Вы находитесь на странице: 1из 197

Problem 13.1 The position of point P relative to point O is given as a function of time by s = 40 + 2t 3 ft.

Determine the position, velocity, and acceleration of the point at t = 4 s. Strategy: Use Eqs. (13.3) and (13.4) to determine the velocity and acceleration as functions of time. Solution:
s = 40 + 2t 3 ft v= ds = 6t 2 ft/s dt s(4s) = 168 ft v(4s) = 96 ft/s a(4s) = 48 ft/s

O s

dv = 12t ft/s2 a= dt

Problem 13.2 The milling machine is programmed so that the position of its head is given as a function of time by s = 0.3 0.2 cos(5t) m. (When t is in seconds, the argument of the cosine is in radians.) (a) (b) Determine the velocity of the head as a function of time. What is the acceleration of the head at t = 4 s?
s

Solution:
s = 0.3 0.2 cos(5t) v= a= ds = sin(5t) dt dv = 5 cos(5t) dt v = sin(5t)m a(4s) = 2.04 m/s2

Problem 13.3 The person drops a ball at time t = 0. The balls position relative to the oor is given as a function of time by s = 16.1t 2 + 4 ft. (a) (b) (c) How long does it take the ball to fall to the oor? What is the balls velocity just before it hits the oor? Determine the balls acceleration.

Solution:
s = 16.1t 2 + 4 ds v= = 32.2t dt a = 32.2 a) 0 = 16.1t 2 + 4 t = 0.498 s b) v(0.498s) = 16.05 ft/s c) a = 32.2 ft/s2
s0

Problem 13.4 The boats position during the interval of time from t = 2 s to t = 10 s is given by s = 4t + 1.6t 2 0.08t 3 m. (a) (b) Determine the boats velocity and acceleration at t = 4 s. What is the boats maximum velocity during this interval of time, and when does it occur?

Solution:
s = 4t + 1.6t 2 0.08t 3 v= a= ds = 4 + 3.2t 0.24t 2 dt dv = 3.2 0.48t dt a) v(4s) = 12.96 m/s2 a(4s) = 1.28 m/s2 b) a = 3.2 0.48t = 0 t = 6.67s v(6.67s) = 14.67 m/s

Problem 13.5 The rocket starts from rest at t = 0 and travels straight up. Its height above the ground can be approximated by the function s = bt 2 + ct 4 , where b and c are constants. At t = 10 s, the rockets velocity and acceleration are v = 229 m/s and a = 28.2 m/s2 . What are its velocity and acceleration at t = 5 s? Solution:
s = bt 2 + ct 4 v = 2bt + 4ct 3 a = 2b + 12ct 2 At t = 10 s (1) (2)

v = 229 m/s = 20b + 4000 c a = 28.2 m/s2 = 2b + 1200 c Solving, we get b = 10.125 and c = 0.006625

substituting these into (1) and (2) and evaluating at t = 5 s, we get v = 104.6 m/s a = 22.2 m/s2

Problem 13.6 The position of a point during the inter1 3 val of time from t = 0 to t = 6 s is given by s = 2 t + 2 6t + 4t m. (a) (b) What is the maximum velocity during this interval of time, and at what time does it occur? What is the acceleration when the velocity is a maximum?
dv = 0 (it could be a minimum) dt da This occurs at t = 4 s. At this point = 3 so we have a maximum. dt Maximum velocity occurs where a = (a) Max velocity is at t = 4 s. where v = 28 m/s and (b) a = 0 m/s2

Solution:
3 2 s = 1 2 t + 6t + 4t m 2 v = 3 2 t + 12t + 4 m/s

a = 3t + 12 m/s2

Problem 13.7 The position of a point during the interval of time from t = 0 to t = 3 seconds is s = 12 + 5t 2 t 3 ft. (a) (b) What is the maximum velocity during this interval of time, and at what time does it occur? What is the acceleration when the velocity is a maximum?
d2v = 6 < 0. The maximum dt 2

Solution:
(a) ds = 10t 3t 2 . The maximum occurs when The velocity is dt dv = 10 6t = 0, from which dt t= 10 = 1.667 seconds. 6 This is indeed a maximum, since velocity is v = 10t 3t 2
t =1.667

= 8.33 ft/s

(b)

The acceleration is

dv = 0 when the velocity is a maximum. dt

Problem 13.8 The rotating crank causes the position of point P as a function of time to be s = 0.4 sin (2 t) m. (a) (b) (c) Determine the velocity and acceleration of P at t = 0.375 s. What is the maximum magnitude of the velocity of P ? When the magnitude of the velocity of P is a maximum, what is the acceleration of P ?
s

Solution:
s = 0.4 sin(2 t) a) v(0.375s) = 1.777 m/s ds v= = 0.8 cos(2 t) dt dv = 1.6 2 sin(2 t) a= dt a(0.375) = 11.2 m/s2 b) vmax = 0.8 = 2.513 m/s2 c) vmax t = 0, n a = 0

Problem 13.9 For the mechanism in Problem 13.8, draw graphs of the position s , velocity v , and acceleration a of point P as functions of time for 0 t 2 s. Using your graphs, conrm that the slope of the graph of s is zero at times for which v is zero, and the slope of the graph of v is zero at times for which a is zero. Solution:

Problem 13.10 A seismograph measures the horizontal motion of the ground during an earthquake. An engineer analyzing the data determines that for a 10-s interval of time beginning at t = 0, the position is approximated by s = 100 cos(2 t) mm. What are (a) the maximum velocity and (b) maximum acceleration of the ground during the 10-s interval? Solution:
(b) (a) The velocity is ds = (2 )100 sin(2 t) mm/s = 0.2 sin(2 t) m/s. dt The velocity maxima occur at dv = 0.4 2 cos(2 t) = 0, dt from which 2 t = (2n 1) (2n 1) , or t = , 2 4 where (2M 1) 10 seconds. 4 d2s = 0.4 2 cos(2 t). dt 2 The acceleration maxima occur at d2v d3s = 2 = 0.8 3 sin(2 t) = 0, dt 3 dt from which 2 t = n , or t = K 10 seconds. 2 These acceleration maxima have the absolute value dv dt
2 2 n = 0.4 = 3.95 m/s . 2

The acceleration is

n , n = 0, 1, 2, . . . K , where 2

n = 1, 2, 3, . . . M,

These velocity maxima have the absolute value ds dt = [0.2 ] = 0.628 m/s.

t=

t=

(2n1) 4

Problem 13.11 During an assembly operation, the robots arm moves along a straight horizontal line. During an interval of time from t = 0 to t = 1 s, the position of the arm is given by s = 30t 2 20t 3 mm. (a) Determine the maximum velocity during this interval of time. (b) What are the position and acceleration when the velocity is a maximum?

Solution:
s = 30t 2 20t 3 mm v = 60t 60t 2 mm/s a = 60 120t mm/s2 da = 120 mm/s3 dt (a) dv Maximum velocity occurs when = a = 0. This occurs at dt 0 = 60 120t or t = 1/2 second. (since da/dt < 0, we have a maximum). The velocity at this time is (b) v = (60) 1 2 60 1 4 mm/s

v = 15 mm/s The position and acceleration at this time are s = 7.5 2.5 mm s = 5 mm a = 0 mm/s2

Problem 13.12 In the test of a prototype car, the driver starts the car from rest at t = 0, accelerates, and then applies the brakes. Engineers measuring the position of the car nd that from t = 0 to t = 18 s the position is 1 1 approximated by s = 5t 2 + t 3 t 4 ft. 3 50 (a) (b) What is the maximum velocity, and at what time does it occur? What is the maximum acceleration, and at what time does it occur?

Solution: Assume that s is measured in feet, and that t is measured


in seconds. (a) The velocity is 2 3 ds = v = 10t + t 2 t ft/s. dt 25 The maximum occurs at dv 6 2 = 10 + 2t t = 0. dt 25 In canonical form this quadratic equation is t 2 + 2bt + c = 0, where b = 25 125 , c= , 6 3 (b)

The maximum velocity is v = 10t + t 2 Acceleration is a= dv = 10 + 2t dt 6 t 2. 25 2 3 t 25 = 125.8 ft/s.

t =11.85

The maximum occurs at da 12 =2 t = 0, dt 25 from which t= 50 = 4.17 seconds. 12

This is a maximum since 12 d2a < 0. = dt 2 t =4.17 25 The maximum acceleration is a = 10 + 2t 6 2 t 25 = 14.17 ft/s2 .

with the solution t = b b2 c = 11.85 or 3.52 seconds. The negative value is outside the interval of interest. At t = 11.85 s there is a maximum since 12 d2v t = 2 dt 2 t =11.85 25 = 3.69 < 0.

t =11.85

t =4.17

Problem 13.13 Suppose you want to approximate the position of a vehicle you are testing by the power series s = A + Bt + Ct 2 + Dt 3 , where A, B , C , and D are constants. The vehicle starts from rest at t = 0 and s = 0. At t = 4 s, s = 176 ft, and at t = 8 s, s = 448 ft. (a) (b) Determine A, B , C , and D . What are the approximate velocity and acceleration of the vehicle at t = 8 s?
The velocity at t = 8 seconds is v = 30t 3t 2
t =8

Solution:
(a) At t = 0, s = 0, hence 0 = A , and s = Bt + Ct 2 + Dt 3 . Since the vehicle starts from rest at t = 0, the velocity is zero, and 0 = B . At t = 4 seconds, s = 176 ft, from which 176 = 16C + 64D . At t = 8 seconds s = 448 ft, from which 448 = 64C + 512D . Solve these two simultaneous equations to obtain C = 15 and D = 1 . (b) The position is given by s = 15t 2 t 3 , from which the velocity is v= ds = 30t 3t 2 . dt

= 48 ft/s .

The acceleration is a= dv = 30 6t. dt

The acceleration at t = 8 seconds is a = [30 6t ]t =8 = 18 ft/s2 .

Problem 13.14 The acceleration of a point is a = 20t m/s2 . When t = 0, s = 40 m and v = 10 m/s. What are the position and velocity at t = 3 s? Solution: The velocity is
v= a dt + C1 , At t = 0, s = 40 m, thus C2 = 40. The position is s= 10 3 t 10t + 40 m. 3

where C1 is the constant of integration. Thus v= 20t dt + C1 = 10t 2 + C1 .

At t = 3 seconds, s= 10 3 t 10t + 40 3 = 100 m.

t =3

At t = 0, v = 10 m/s, hence C1 = 10 and the velocity is v = 10t 2 10 m/s. The position is s= v dt + C2 ,

The velocity at t = 3 seconds is v = 10t 2 10


t =3

= 80 m/s .

where C2 is the constant of integration. s= (10t 2 10) dt + C2 = 10 3 t 10t + C2 . 3

Problem 13.15 The acceleration of a point is a = 60t 36t 2 ft/s2 . When t = 0, s = 0 and v = 20 ft/s. What are position and velocity as a function of time? Solution: The velocity is
v= a dt + C1 = (60t 36t 2 ) + C1 = 30t 2 12t 3 + C1 . The position is s= v dt + C2 = (30t 2 12t 3 + 20) + C2

At t = 0, v = 20 ft/s, hence C1 = 20, and the velocity as a function of time is v = 30t 2 12t 3 + 20 ft/s .

= 10t 3 3t 4 + 20t + C2 . At t = 0, s = 0, hence C2 = 0, and the position is s = 10t 3 3t 4 + 20t ft

Problem 13.16 The snow petrel takes off with constant acceleration. If it requires a distance s = 4 m and is moving at 6 m/s when it lifts off, how much time does its takeoff require?

Solution:
a = constant v = at + v o 0 s = at 2 /2 + s o 0 when s = 4 m, v = 6 m/s 4 = at 2 /2 6 = at 4 = (at)t/2 = 6 t/2 t = 1.33 s
s

Problem 13.17 A bioengineer studying the mechanics of bird ight models the acceleration of the snow petrel shown in Problem 13.16 by an equation of the form a = C + Dt , where C and D are constants. Measurements obtained from videotape indicate that one bird requires 1.42 seconds and a distance of 4.3 m to take off and is moving at 6.1 m/s when it lifts off. What are the constants C and D ? Solution: s0 = v0 = 0
a v s = C + Dt = Ct + Dt 2 /2 = Ct 2 /2 + Dt 3 /6 Integrating twice We have two equations in two unknowns. Solving, we get C = 4.20 m/s2 D = 0.130 m/s3

Substituting the known numbers, we get 6.1 = C(1.42) + D(1.42)2 /2 4.3 = C(1.42)2 /2 + D(1.42)3 /6

Problem 13.18 Missiles designed for defense against ballistic missiles have attained accelerations in excess of 100 g s, or 100 times the acceleration due to gravity. Suppose that a missile lifts off from the ground and has a constant vertical acceleration of 100 g s. (a) (b) How long does it take to reach a velocity of 60 mi/h? How long does it take to reach an altitude of 10,000 ft? How fast is it going when it reaches that altitude?

U S A R M Y

Solution:
a = 100g = 100(32.2 ft/s2 ) = 3220 ft/s2 v = (3220 ft/s2 )t s = (1610 ft/s2 )t 2 a) b) 60 mph 88 ft/s 60 mph = (3220 ft/s2 )t t = 0.0273 s

10,000 ft = (1610 ft/s2 )t 2 t = 2.49 s v(2.49 s) = (3220 ft/s2 )(2.49 s) = 8025 ft/s

Problem 13.19 The acceleration due to gravity at sea level on earth is 9.81 m/s2 and the acceleration due to gravity at the surface of the moon is 1.62 m/s2 . (a) (b) If an object at sea level on the earth is given an upward velocity of 10 m/s and aerodynamic drag is negligible, how high does it go? If the object is at the surface of the moon and is given an upward velocity of 10 m/s, how high does it go?

Solution:
a = g v = gt + v0 1 s = gt 2 + v0 t 2 Searth = (10 m/s)2 2(9.81 m/s2 ) (10 m/s)2 2(1.62 m/s2 ) v = gt + v0 = 0 t = 1 s= g 2 = 5.10 m = 30.9 m v0 g
2

v0 g = v0 2 2g

+ v0

v0 g

a)

b) Smoon =

Problem 13.20 The airplane releases its drag parachute at time t = 0. Its velocity is given as a function of time by 80 v= m/s. 1 + 0.32t What is the airplanes acceleration at t = 3 s?

Solution:
v= 80 dv 25.6 a(3 s) = 6.66 m/s2 ; a= = 1 + 0.32t dt (1 + 0.32t)2

Problem 13.21 How far does the airplane in Problem 13.20 travel during the interval of time from t = 0 to t = 10 s?

Solution:
v= 80 ; s= 1 + 0.32t
10 s 0

1 + 3.2 80 dt = 250 ln = 359 m 1 + 0.32t 1

Problem 13.22 The velocity of a bobsled is v = 10t ft/s. When t = 2 s, the position is s = 25 ft. What is its position at t = 10 s?

Solution: The equation for straight line displacement under constant acceleration is
s= a(t t0 )2 + v(t0 )(t t0 ) + s(t0 ). 2

Choose t0 = 0. At t = 2, the acceleration is a= dv(t) dt = 10 ft/s2 ,

t =2

the velocity is v(t0 ) = 10(2) = 20 ft/s, and the initial displacement is s(t0 ) = 25 ft. At t = 10 seconds, the displacement is s= 10 (10 2)2 + 20(10 2) + 25 = 505 ft 2

Problem 13.23 In September, 2003, Tony Schumacher started from rest and drove a quarter mile (1320 ft) in 4.498 seconds in a National Hot Rod Association race. His speed as he crossed the nish line was 328.54 mi/h. Assume that the cars acceleration can be expressed by a linear function of time a = b + ct . (a) (b) Determine the constants b and c. What was the cars speed 2 s after the start of the race?

Solution:
a = b + ct, v = bt + ct 2 , 2 s= bt 2 ct 3 + 2 6

Both constants of integration are zero. (a) 328.54 mph 88 ft/s c = b(4.498 s) + (4.498 s)2 60 mph 2

1320 ft =

b c (4.498 s)2 + (4.498 s)3 2 6 b = 177 ft/s2 c = 31.16 ft/s3 c (2 s)2 = 292 ft/s 2

(b)

v = b(2 s) +

Problem 13.24 The velocity of an object is v = 200 2t 2 m/s. When t = 3 seconds, its position is s = 600 m. What are the position and acceleration of the object at t = 6 s? Solution: The acceleration is
dv(t) = 4t m/s2 . dt At t = 6 seconds, the acceleration is a = 24 m/s2 . Choose the initial conditions at t0 = 3 seconds. The position is obtained from the velocity:
6 3

s(t t0 ) =

v(t) dt + s(t0 ) = 200t

2 3 t 3

+ 600 = 1070 m .
3

Problem 13.25 An inertial navigation system measures the acceleration of a vehicle from t = 0 to t = 6 s and determines it to be a = 2 + 0.1t m/s2 . At t = 0, the vehicles position and velocity are s = 240 m, v = 42 m/s. What are the vehicles position and velocity at t = 6 s? Solution:
a = 2 + 0.1t m/s2 v0 = 42 m/s s0 = 240 m Integrating v = v0 + 2t + 0.1t 2 /2 s = v0 t + t 2 + 0.1t 3 /6 + s0 Substituting the known values at t = 6 s, we get v = 55.8 m/s s = 531.6 m

Problem 13.26 The missile shown in Problem 13.18 lifts off and accelerates for 3 s at 100g s. After 3 s, its weight and aerodynamic drag cause it to have a nearly constant deceleration of 4g s. How long does it take the missile to go from the ground to an altitude of 15 km (approximately 50,000 ft)? Solution: (0 t 3 s) v0 = s0 = 0 a = 981 m/s2
a = 981 m/s2 v = 981t m/s s= At 981t 2 2 m = 490.5t 2 We must solve for v0 and s0 . Note, v0 and s0 are not the velocity and acceleration at t = 3 s. They are values that correspond to a constant 4g deceleration starting at t = 0 which ies through the conditions v = 2943 m/s, s = 4415 m at t = 3 s. Thus, 2943 = v0 4g(3) 4415 = s0 + v0 (3) 4g(3)2 /z Solving, v0 = 3061 m/s, s0 = 4591 m. We now need to nd when s = 15000 m, we have s = 4591 + 3061t 4gt 2 /z and s = 15000 m. Solving, we get t = 6.69 seconds
2

t = 3 s, s = 4415 m

v = 2943 m/s, For 3 t;

a = 39.24 m/s

a = 39.24 m/s2 v = v0 39.24t m/s s = s0 + v0 t 39.24t 2 /2 m

Problem 13.27 The graph shows the airplanes acceleration during its takeoff. What is the airplanes velocity when it rotates (lifts off) at t = 30 s?

a 9 ft/s2

3 ft/s2 0 t 0 5s 30 s

Solution: Velocity = Area under the curve


v= 1 (3 ft/s2 + 9 ft/s2 )(5 s) + (9 ft/s2 )(25 s) = 255 ft/s 2

Problem 13.28 Determine the distance traveled during its takeoff by the airplane in Problem 13.27. Solution: for 0 t 5 s
a= 6 ft/s2 5s 6 ft/s2 5s t + (3 ft/s2 ), v= 6 ft/s2 5s t2 + (3 ft/s2 )t 2

s=

t3 t2 + (3 ft/s2 ) 6 2

v(5 s) = 30 ft/s, s(5 s) = 62.5 ft for 5 s t 30 s a = 9 ft/s2 , s = (9 ft/s2 ) v = (9 ft/s2 )(t 5 s) + 30 ft/s, (t 5 s)2 + (30 ft/s)(t 5 s) + 62.5 ft 2

s(30 s) = 3625 ft

Problem 13.29 The car is traveling at 30 mi/h when the trafc light 295 ft ahead turns yellow. The driver takes one second to react before he applies the brakes. (a) (b) After he applies the brakes, what constant rate of deceleration will cause the car to come to a stop just as it reaches the light? How long does it take the car to travel the 295 ft?

30 mi/h

295 ft

Solution: for 0 t 1 s
a = 0, v = 30 mph s(1 s) = 44 ft for t > 1 s a = c (constant), v = ct + 44 ft/s, s = c At the stop we have 295 ft = c t2 + (44 ft/s)t + 44 ft 2 c = 3.857 ft/s2 t = 11.41 s t2 + (44 ft/s)t + 44 ft 2 88 ft/s 60 mph = 44 ft/s, s = (44 ft/s)t

0 = ct + 44 ft/s

a) b)

Problem 13.30 A t = 0 a motorist traveling at 100 km/h sees a deer standing in the road 100 m ahead. After a reaction time of 0.3 seconds, he applies the brakes and decelerates at a constant rate of 4 m/s2 . If the deer takes 5 seconds from t = 0 to react and leave the road, does the motorist miss it? Solution: Divide the time into two intervals, the reaction time of the motorist (0.3 s) and the time before the deer leaves the road (5 0.3 = 4.7 s). The initial velocity is
v(0) = 100 km = 27.8 m/s. h

The distance traveled in the rst interval is s = 27.8(0.3) = 8.33 m. The distance traveled in the second interval is a s(t) = t 2 + v(0)t = 2(4.7)2 + 27.8(4.7) = 86.4 m. 2 The total distance traveled is s(5) = 8.33 + 86.4 = 94.7 m, which is less than the 100 m from motorist to deer at t = 0. Yes, the motorist misses the deer.

Problem 13.31 A high-speed rail transportation system has a top speed of 100 m/s. For the comfort of the passengers, the magnitude of the acceleration and deceleration is limited to 2 m/s2 . Determine the time required for a trip of 100 km. Strategy: A graphical approach can help you solve this problem. Recall that the change in the position from an initial time t0 to a time t is equal to the area dened by the graph of the velocity as a function of time from t0 to t . Solution: Divide the time of travel into three intervals: The time required to reach a top speed of 100 m/s, the time traveling at top speed, and the time required to decelerate from top speed to zero. From symmetry, the rst and last time intervals are equal, and the distances traveled during these intervals are equal. The initial time is obtained from v(t1 ) = at1 , from which t1 = 100/2 = 50 s. The distance trav2 /2 from which s(t ) = 2(50)2 /2 = eled during this time is s(t1 ) = at1 1 2500 m. The third time interval is given by v(t3 ) = at3 + 100 = 0, from which t3 = 100/2 = 50 s. Check. The distance traveled is s(t3 ) = a 2 t3 + 100t3 , from which s(t3 ) = 2500 m. Check. The distance trav2 eled at top speed is s(t2 ) = 100000 2500 2500 = 95000 m = 95 km. The time of travel is obtained from the distance traveled at zero acceleration: s(t2 ) = 95000 = 100t2 , from which t2 = 950. The total time of travel is ttotal = t1 + t2 + t3 = 50 + 950 + 50 = 1050 s
= 17.5 minutes . A plot of velocity versus time can be made and the area under the curve will be the distance traveled. The length of the constant speed section of the trip can be adjusted to force the length of the trip to be the required 100 km.

Problem 13.32 The nearest star, Proxima Centauri, is 4.22 light years from the Earth. Ignoring relative motion between the solar system and Proxima Centauri, suppose that a spacecraft accelerates from the vicinity of the Earth at 0.01 g (0.01 times the acceleration due to gravity at sea level) until it reaches one-tenth the speed of light, coasts until time to decelerate, then decelerates at 0.01 g until it comes to rest in the vicinity of Proxima Centauri. How long does the trip take? (Light travels at 3 108 m/s. A solar year is 365.2422 solar days.) Solution: The distance to Proxima Centauri is
d = (4.22 light - year)(3 108 m/s)(365.2422 day) = 3.995 1016 m. Divide the time of ight into the three intervals. The time required to reach 0.1 times the speed of light is t1 = v 3 107 m/s = 3.0581 108 seconds. = a 0.0981 m/s2 86400 s 1 day where v(0) = 0 and s(0) = 0 (from the conditions in the problem), from which s(t1 ) = 4.587 1015 m. From symmetry, t3 = t1 , and s(t1 ) = s(t3 ). The length of the middle interval is s(t2 ) = d s(t1 ) s(t3 ) = 3.0777 1016 m. The time of ight at constant velocity is t2 = 3.0777 1016 m = 1.026 109 seconds. 3 107

The total time of ight is ttotal = t1 + t2 + t3 = 1.63751 109 seconds. In solar years: ttotal = 1.63751 109 sec = 51.9 solar years 1 solar years 365.2422 days 1 days 86400 sec

The distance traveled is s(t1 ) = a 2 t + v(0)t + s(0), 2 1

Problem 13.33 A race car starts from rest and accelerates at a = 5 + 2t ft/s2 for 10 seconds. The brakes are then applied, and the car has a constant acceleration a = 30 ft/s2 until it comes to rest. Determine (a) the maximum velocity, (b) the total distance traveled; (c) the total time of travel. Solution:
(a) For the rst interval, the velocity is v(t) = (5 + 2t) dt + v(0) = 5t + t 2 (c) the total time of travel is t2 = 15. The total distance traveled is s(t2 10) = a (t2 10)2 + v(10)(t2 10) + s(10), 2

from which (b) s(5) = 30 2 5 + 150(5) + 583.33 = 958.33 ft 2

since v(0) = 0. The velocity is an increasing monotone function; hence the maximum occurs at the end of the interval, t = 10 s, from which vmax = 150 ft/s . (b) The distance traveled in the rst interval is s(10) =
10 0 (5t

+ t 2 ) dt =

5 2 1 3 t + t 2 3

10

= 583.33 ft.
0

The time of travel in the second interval is v(t2 10) = 0 = a(t2 10) + v(10), t2 10 s, from which (t2 10) = 150 = 5, 30 and

Problem 13.34 When t = 0, the position of a point is s = 6 m and its velocity is v = 2 m/s. From t = 0 to t = 6 s, the acceleration of the point is a = 2 + 2t 2 m/s2 . From t = 6 s until it comes to rest, its acceleration is a = 4 m/s2 . (a) (b) What is the total time of travel? What total distance does the point move?
The total time of travel is (a) (b) ttotal = 39.5 + 6 = 45.5 seconds. The distance traveled is s(t 6) = 4 (t 6)2 + v(6)(t 6) + s(6) 2

Solution: For the rst interval the velocity is


v(t) = (2 + 2t 2 ) dt + v(0) = 2t + 2 3 t + 2 m/s. 3

The velocity at the end of the interval is v(6) = 158 m/s. The displacement in the rst interval is s(t) = 2t + 2 3 t +2 3 dt + 6 = t 2 + 1 4 t + 2t + 6. 6

= 2(39.5)2 + 158(39.5) + 270, from which the total distance is stotal = 3390 m

The displacement at the end of the interval is s(6) = 270 m. For the second interval, the velocity is v(t 6) = a(t 6) + v(6) = 0, t 6, from which (t 6) = v(6) 158 = = 39.5. a 4

Problem 13.35 Zoologists studying the ecology of the Serengeti Plain estimate that the average adult cheetah can run 100 km/h and that the average springbuck can run 65 km/h. If the animals run along the same straight line, start at the same time, and are each assumed to have constant acceleration and reach top speed in 4 s, how close must the a cheetah be when the chase begins to catch a springbuck in 15 s? Solution: The top speeds are Vc = 100 km/h = 27.78 m/s for the cheetah, and Vs = 65 km/h = 18.06 m/s. The acceleration is ac = Vc Vs = 6.94 m/s2 for the cheetah, and as = = 4.513 m/s2 for the 4 4 springbuck. Divide the intervals into the acceleration phase and the chase phase. For the cheetah, the distance traveled in the rst is 6.94 2 sc (t) = (4) = 55.56 m. The total distance traveled at the end 2 of the second phase is stotal = Vc (11) + 55.56 = 361.1 m. For the springbuck, the distance traveled during the acceleration phase is 4.513 2 ss (t) = (4) = 36.11 m. The distance traveled at the end of the 2 second phase is ss (t) = 18.06(11) + 36.1 = 234.7 m. The permissible separation between the two at the beginning for a successful chase is d = sc (15) ss (15) = 361.1 234.7 = 126.4 m

Problem 13.36 Suppose that a person unwisely drives 75 mi/h in a 55 mi/h zone and passes a police car going 55 mi/h in the same direction. If the police begin constant acceleration at the instant they are passed and increase their speed to 80 mi/h in 4 s, how long does it take them to be even with the pursued car? Solution: The conversion from mi/h to ft/s is
1.467 ft h mi second .

The acceleration of the police car is a= (80 55)(1.467) ft/s = 9.169 ft/s2 . 4s

The distance traveled during acceleration is s(t1 ) = 9.169 2 (4) + 55(1.467)(4) = 396 ft. 2

The distance traveled by the pursued car during this acceleration is sc (t1 ) = 75(1.467)t1 = 110(4) = 440 ft. The separation between the two cars at 4 seconds is d = 440 396 = 44 ft. This distance is traversed in the time t2 = 44 = 6. (80 75)(1.467)

The total time is ttotal = 6 + 4 = 10 seconds.

Problem 13.37

If = 1 rad and

is the velocity of P relative to O ?

d = 1 rad/s, what dt
2 ft 2 ft

Strategy: The position of P relative to O as s = (2 ft) cos + (2 ft) cos and take the derivative of this expression with respect to time to determine the velocity.
O

P s

Solution: The distance s from point O is


s = (2 ft) cos + (2 ft) cos . The derivative is ds d = 4 sin . dt dt For = 1 radian and d = 1 radian/second, dt

ds = v(t) = 4(sin(1 rad)) = 4(0.841) = 3.37 ft/s dt

Problem 13.38 In Problem 13.37, if = 1 rad, d /dt = 2 rad/s and d 2 /dt 2 = 0, what are the velocity and acceleration of P relative to O ? Solution: The velocity is
ds d = 4 sin = 4(sin(1 rad))(2) = 6.73 ft/s . dt dt The acceleration is d2s = 4 cos dt 2 from which d2s = a = 4 cos(1 rad)(4) = 8.64 ft/s2 dt 2 d dt
2

4 sin

d2 dt 2

Problem 13.39

d = 1 rad/s, what dt is the velocity and acceleration of P relative to O ? If = 1 rad and


200 mm

400 mm

Solution: The acute angle formed by the 400 mm arm with the horizontal is given by the sine law:
200 400 = , sin sin from which sin = 200 sin . 400

O P s

For = 1 radian, = 0.4343 radians. The position relative to O is. s = 200 cos + 400 cos . The velocity is ds = v(t) = 200 sin dt d dt 400 sin d dt .

From the expression for the angle , cos d dt = 0.5 cos d dt ,

from which the velocity is v(t) = (200 sin 200 tan cos ) Substitute: v(t) = 218.4 mm/s . d dt .

Problem 13.40 An engineer designing a system to control a router for a machining process models the system so that the routers acceleration during an interval of time is given by a = 0.4v in/s2 , where v is the velocity of the router in in/s. When t = 0, the position is s = 0 and the velocity is v = 2 in/s. What is the velocity at t = 3 s?
s

Solution:
a=
v 2 in./s

dv = (0.4 s1 )v dt dv = v
3s 0

(0.4 s1 )dt ln

v 2 in./s

= 1.2

v = (2 in./s)e1.2 = 0.602 in./s

Problem 13.41 What is the position of the router in Problem 13.40 at t = 3 s? Solution:
a=
v 2 in./s

dv = (0.4 s1 )v dt dv = v
t 0

(0.4 s1 )dt ln

v 2 in./s

= (0.4 s1 )t

v=

ds 1 = (2 in./s)e(0.4 s )t dt
3s 0

s(3 s) =

(2 in./s)e(0.4 s

1 )t

dt = 3.494 in.

Problem 13.42 The boat is moving at 10 m/s when its engine is shut down. Due to hydrodynamic drag, its subsequent acceleration is a = 0.05v 2 m/s2 , where v is the velocity of the boat in m/s. What is the boats velocity 4 s after the engine is shut down? Solution:
a=
v 10 m/s

dv = (0.05 m1 )v 2 dt dv = (0.05 m1 ) v2 10 m/s 1 + (0.5 s1 )t


t 0

dt

1 v

v 10 m/s

= (0.05 m1 )t

v=

v(4 s) = 3.33 m/s

Problem 13.43 In Problem 13.42, what distance does the boat move in the 4 s following the shutdown of its engine? Solution: From Problem 13.42 we know
v= 10 m/s ds = s(4 s) = dt 1 + (0.5 s1 )t 2 + (1 s1 )(4 s) 2
4 s 0

10 m/s 1 + (0.5 s1 )t

dt

s(4 s) = (20 m) ln

= 21.97 m

Problem 13.44 A steel ball is released from rest in a container of oil. Its downward acceleration is a = 2.4 0.6v in/s2 , where v is the balls velocity in in/s. What is the balls downward velocity 2 s after it is released? Solution:
a=
v 0

dv = (2.4 in./s) (0.6 s1 )v dt dv (2.4 in./s) (0.6 s


1

)v

=
0

dt

v + 4 in./s 5 ln 3 4 in./s

= t v = (4 in./s) 1 e(0.6

s1 )t

v(2 s) = 2.795 in./s

Problem 13.45 In Problem 13.44, what distance does the ball fall in the rst 2 s after its release? Solution: From 13.44 we know
v= ds = (4 in./s) 1 e(0.6 dt
t 0 s1 )t

s(2 s) =

(4 in./s) 1 e(0.6
s1 )t

s1 )t

dt

20 in. (0.6 e 3

1 + (4 in./s)t

s(2 s) = 3.34 in.

Problem 13.46 The greatest ocean depth yet discovered is the Marianas Trench in the western Pacic Ocean. A steel ball released at the surface requires 64 minutes to reach the bottom. The balls downward acceleration is a = 0.9g cv , where g = 9.81 m/s2 and the constant c = 3.02 s1 . What is the depth of the Marianas Trench in kilometers? Solution:
a= dv = 0.9g cv. dt Integrating,
s 0

ds =
0

0.9g (1 ect ) dt. c

Separating variables and integrating,


v 0

We obtain s= 0.9g c t+ ect 1 . c c

dv = 0.9g cv

t 0

dt = t.

Integrating and solving for v , v= 0.9g ds = (1 ect ). dt c

At t = (64)(60) = 3840 s, we obtain s = 11,225 m.

Problem 13.47 The acceleration of a regional airliner during its takeoff run is a = 14 0.0003v 2 ft/s2 , where v is its velocity in ft/s. How long does it take the airliner to reach its takeoff speed of 200 ft/s? Solution:
a= dv = (14 ft/s2 ) (0.0003 ft1 )v 2 dt dv (14 ft/s2 ) (0.0003 ft1 )v 2 =
0 t

200 ft/s 0

dt

t = 25.1 s

Problem 13.48 In Problem 13.47, what distance does the airliner require to take off? Solution:
a=v dv = (14 ft/s2 ) (0.0003 ft1 )v 2 ds vdv (14 ft/s ) (0.0003 ft
2 1

200 ft/s 0

)v 2

=
0

ds

s = 3243 ft

Problem 13.49 A sky diver jumps from a helicopter and is falling straight down at 30 m/s when her parachute opens. From then on, her downward acceleration is approximately a = g cv 2 , where g = 9.81 m/s2 and c is a constant. After an initial transient period she descends at a nearly constant velocity of 5 m/s. (a) (b) (c) What is the value of c, and what are its SI units? What maximum deceleration is the sky diver subjected to? What is her downward velocity when she has fallen 2 meters from the point where her parachute opens?

Solution: Assume c > 0.


(a) After the initial transient, she falls at a constant velocity, so that the acceleration is zero and cv 2 = g , from which c= (b) g 9.81 m/s2 = = 0.3924 m1 v2 (5)2 m2 /s2

Integrate: 1 2c ln |g cv 2 | = s + C.

When the parachute opens s = 0 and v = 30 m/s, from which C= 1 2c ln |g 900c| = 7.4398.

The maximum acceleration (in absolute value) occurs when the parachute rst opens, when the velocity is highest: amax = |g cv 2 | = |g c(30)2 | = 343.4 m/s2

The velocity as a function of distance is ln |g cv 2 | = 2c(s + C). For s = 2 m, v = 14.4 m/s

(c)

Choose coordinates such that distance is measured positive downward. The velocity is related to position by the chain rule: dv dv ds dv = =v = a, dt ds dt ds from which v dv = ds. g cv 2

Problem 13.50 The rocket sled starts from rest and accelerates at a = 30 + 2t m/s2 until its velocity is 400 m/s. It then hits a water brake and its acceleration is a = 0.003v 2 m/s2 until its velocity decreases to 100 m/s. What total distance does the sled travel? Solution: Acceleration Phase
a = 30 + 2t m/s2 v = 30t + t 2 m/s s = 15t 2 + t 3 /3 m When v = 400 m/s, acceleration ends. At this point, t = 10 s and s = 1833 m. Deceleration Phase starts at s1 = 1833 m, v1 = 400 m/s. Let us start a new clock for the deceleration phase. vf = 100 m/s a=v
sf s1

dv = 0.003v 2 ds 1 (0.003)
vf v1

ds =

v dv v2

sf 1833 m = sf = 2300 m

1 [ln(100) ln(400)] (0.003)

Problem 13.51 In Problem 13.50, what is the sleds total time of travel? Solution: From the solution to Problem 13.50, the acceleration takes 10 s. At t = 10 s, the velocity is 400 m/s. We need to nd out how long it takes to decelerate from 400 m/s to 100 m/s and add this to the 10 s required for acceleration. The deceleration is given as
a=
td 0

dv = 0.003v 2 m/s2 dt dt =
100 400

0.003

dv v2 = 1 1 100 400

0.003td =

1 v

100 400

0.003td = td = 2.5 s

3 400

t = 10 + td = 12.5 s

Problem 13.52 A cars acceleration is related to its position by a = 0.01s m/s2 . When s = 100 m, the car is moving at 12 m/s. How fast is the car moving when s = 420 m? Solution:
a=v
vf 12

dv = 0.01s m/s2 ds
420

v dv = 0.01

s ds
100 420 m 100 m

v2 2

vf

= 0.01
12 m/s 2 vf

s2 2

122 (4202 1002 ) + 0.01 2 2

vf = 42.5 m/s

Problem 13.53 Engineers analyzing the motion of a linkage determine that the velocity of an attachment point is given by v = A + 4s 2 ft/s, where A is a constant. When s = 2 ft, its acceleration is measured and determined to be a = 320 ft/s2 . What is its velocity of the point when s = 2 ft? Solution: The velocity as a function of the distance is
v dv = a. ds When s = 2 ft, a = 320 ft/s2 , from which A = 4. The velocity at s = 2 ft is v = 4 + 4(22 ) = 20 ft/s

Solve for a and carry out the differentiation. a=v dv = (A + 4s 2 )(8s). ds

Problem 13.54 The acceleration of an object is given by the function a = 2s ft/s2 . When t = 0, v = 1 ft/s. What is the velocity when the object has moved 2 ft from its initial position? Solution: The differential equations for the velocity and distance
are dv = 2s, dt and ds = v. dt

Use the chain rule, separate variables and integrate: v dv = 2s, ds

from which C1 = 1 and v 2 = 2s 2 + 1. At s = 2 ft, v 2 = 9, v = 3 ft/s where the positive sign on the square root is chosen because the velocity increases with distance.

Problem 13.55 Gas guns are used to investigate the properties of materials subjected to high-velocity impacts. A projectile is accelerated through the barrel of the gun by gas at high pressure. Assume that the acceleration of the projectile is given by a = c/s m/s2 , where s is the position of the projectile in the barrel in meters and c is a constant that depends on the initial gas pressure behind the projectile. The projectile starts from rest at s = 1.5 m and accelerates until it reaches the end of the barrel at s = 3 m. Determine the value of the constant c necessary for the projectile to leave the barrel with a velocity of 200 m/s. Solution:
a=v dv c = , ds s
200 m/s 0

vdv =

3m 1.5 m

c ds s

(200 m/s)2 3m = c ln 2 1.5 m

c = 28.85 103 m2 /s2

Problem 13.56 If the propelling gas in the gas gun described in Problem 13.55 is air, a more accurate modeling of the acceleration of the projectile is obtained by assuming that the acceleration of the projectile is given by a = c/s m/s2 , where = 1.4 is the ratio of specic heats for air. (This means that an isentropic expansion process is assumed instead of the isothermal process assumed in Problem 13.55.) Determine the value of the constant c necessary for the projectile to leave the barrel with a velocity of 200 m/s.

Solution:
a=v dv c = 1.4 , ds s
200 m/s 0

vdv =

3m 1.5 m

c ds s 1.4

(200 m/s)2 = 2.5c (3m)0.4 (1.5m)0.4 2 c = 38.86 103 m2.4 /s2

Problem 13.57 A spring-mass oscillator consists of a mass and a spring connected as shown. The coordinate s measures the displacement of the mass relative to its position when the spring is unstretched, If the spring is linear, the mass is subjected to a deceleration proportional to s . Suppose that a = 4s m/s2 , and that you give the mass a velocity v = 1 m/s in the position s = 0. (a) (b) How far will the mass move to the right before the spring brings it to a stop? What will be the velocity of the mass when it has returned to the position s = 0?

Solution: The velocity of the mass as a function of its position is given by v dv/ds = a . Substitute the given acceleration, separate variables, and integrate: v dv = 4s ds , from which v 2 /2 = 2s 2 + C . The initial velocity v(0) = 1 m/s at s = 0, from which C = 1/2. The velocity is v 2 /2 = 2s 2 + 1/2.
(a) The velocity is zero at the position given by 0 = 2(s1 )2 + 1 , 2

from which

s1 =

1 1 = m. 4 2

Since the displacement has the same sign as the velocity, s1 = +1/2 m is the distance traveled before the spring brings it to a stop. (b) 2 = 1 m/s. From 2 the physical situation, the velocity on the rst return is negative (opposite the sign of the initial displacement), At the return to s = 0, the velocity is v = v = 1 m/s .

Problem 13.58 In Problem 13.57, suppose that at t = 0 you release the mass from rest in the position s = 1 m. Determine the velocity of the mass as a function of s as it moves from the initial position to s = 0.

Solution: From the solution to Problem 13.57, the velocity as a


function of position is given by v2 = 2s 2 + C. 2 At t = 0, v = 0 and s = 1 m, from which C = 2(1)2 = 2. The velocity is given by
1 v(s) = (4s 2 + 4) 2 = 2 1 s 2 m/s.

From the physical situation, the velocity is negative (opposite the sign of the initial displacement): v = 2 1 s 2 m/s . [Note: From the initial conditions, s 2 1 always.]

Problem 13.59 In Problem 13.57, suppose that at t = 0 you release the mass from rest in the position s = 1 m. Determine the position of the mass as a function of time as it moves from its initial position to s = 0. Solution: The differential equations for the velocity and position are dv = 4s, dt and ds = v. dt ds = 2 dt, cos1 (s) = 2t + C. 1 s2 At t = 0, s = 1 m, from which C = cos1 (1) = 0, from which s(t) = cos 2t m where the negative sign for the square root is chosen because s decreases with increasing t at t = 0 + Separate variables and integrate:

Use the chain rule: v dv = 4s. ds

Separate variables and integrate: v 2 = 4s 2 + C . At t = 0, s = 1 m, and v = 0, from which C = 4, and v 2 = 4(1 s 2 ) Substitute: ds = 2 1 s 2 . dt

Problem 13.60 The mass is released from rest with the springs unstretched. Its downward acceleration is a = 32.2 50s ft/s2 , where s is the position of the mass measured from the position in which it is released. (a) How far does the mass fall? (b) What is the maximum velocity of the mass as it falls?
s

Solution: The acceleration is given by


a= dv dv ds dv = =v = 32.2 50s ft/s2 . dt ds dt ds

Integrating, we get
v 0

v dv =
0

(32.2 50s) ds

or

v2 = 32.2s 25s 2 . 2

(a)

The mass falls until v = 0. Setting v = 0, we get 0 = (32.2 25s)s . We nd v = 0 at s = 0 and at s = 1.288 ft. Thus, the mass falls 1.288 ft before coming to rest. From the integration of the equation of motion, we have v 2 = dv 2(32.2s 25s 2 ). The maximum velocity occurs where = 0. ds dv = From the original equation for acceleration, we have a = v ds 2 (32.2 50s) ft/s . Since we want maximum velocity, we can assume that v = 0 at this point. Thus, 0 = (32.2 50s), or s = (32.2/50) ft when v = vMAX . Substituting this value for s into the equation for v , we get
2 =2 vMAX

(b)

(32.2)2 (25)(32.2)2 50 502

or vMAX = 4.55 ft/s

Problem 13.61 Suppose that the mass in Problem 13.60 is in the position s = 0 and is given a downward velocity of 10 ft/s. (a) (b) How far does the mass fall? What is the maximum velocity of the mass as it falls?
(a) The mass falls until v = 0 0 = (10 ft/s)2 + (64.4 ft/s2 )s (50 s2 )s 2 s = 2.20 ft (b) The maximum velocity occurs when a = 0 0 = (32.2 ft/s2 ) (50 s2 )s s = 0.644 ft v2 2 (10 ft/s)2 2 = (32.2 ft/s2 )s (50 s2 ) s2 2 v 2 = (10 ft/s)2 + (64.4 ft/s2 )(0.644 ft) (50 s2 )(0.644 ft)2 v = 10.99 ft/s

Solution:
a=v
v 10 ft/s

dv = (32.2 ft/s2 ) (50 s2 )s ds vdv =


0 s

[(32.2 ft/s2 ) (50 s2 )s ]ds

v 2 = (10 ft/s)2 + (64.4 ft/s2 )s (50 s2 )s 2

Problem 13.62 If a spacecraft is 100 mi above the surface of the earth, what initial velocity v0 straight away from the earth would be required for the vehicle to reach the moons orbit 238,000 mi from the center of the earth? The radius of the earth is 3960 mi. Neglect the effect of the moons gravity. (See Example 13.5)

100 mi
0

238,000 mi

Solution: For computational convenience, convert the acceleration due to Earths gravity into the units given in the problem, namely miles and hours:
g= 32.17 ft 1 s2 1 mile 5280 ft 36002 s2 1 h2 = 78962.7 mi/h .
2

from which C = 10405562 mi2 /h2 . At the 100 mile altitude, the equation for the velocity is
2 =2 g v0 2 RE RE + 100

+ C.

The velocity as a function of position is given by gR 2 dv v = a = 2E . ds s Separate variables,


2 v dv = gRE

From which v0 = 599575671 = 24,4862 mi/h

Converting: v0 = 24486.2 mi 1h 5280 ft 1 mi 1h 3600 s = 35,913.1 ft/s .

ds . s2

Integrate:
2 v 2 = 2gRE

Check: Use the result of Example 13.5 1 2 + C. 1 1 , s0 H

v0 =

2 2gRE

Suppose that the velocity at the distance of the Moons orbit is zero. Then 0 = 2(78962.7) 39602 238,000 + C,

(where H > s0 always), and H = 238,000, from which v0 = 24,486.2 mi/h. check.

Problem 13.63 The moons radius is 1738 km. The magnitude of the acceleration due to gravity of the moon at a distance s from the center of the moon is 4.89 1012 m/s2 . s2 Suppose that a spacecraft is launched straight up from the moons surface with a velocity of 2000 m/s. (a) (b) What will the magnitude of its velocity be when it is 1000 km above the surface of the moon? What maximum height above the moons surface will it reach?

v0 = 2000 m/s a=v

Solution: Set G = 4.89 1012 m3 /s2 , r0 = 1.738 106 m,


dv G = 2 ds s
v v0

vdv =

r r0

v0 2 1 1 G v2 =G ds s2 2 2 r r0

v 2 = v0 2 + 2G

r0 r rr0

(a) (b)

v(r0 + 1.0 106 m) = 1395 m/s The maximum velocity occurs when v = 0 r= 2G = 6010 km 2G r0 v0 2 h = r r0 = 4272 km

Problem 13.64* The velocity of an object subjected only to the earths gravitational eld is
2 2 v = v0 + 2gRE

1 1 s s0

1/2

where s is position relative to the center of the earth, v0 is the velocity at position s0 , and RE is the earths radius. Using this equation, show that the objects acceleration 2 2 is given as a function of s by a = gRE /s . Solution:
2 + 2gR 2 v = v0 E

1 1 s s0

1/2

a=

dv dv =v dt ds
2 2 2gRE 2gRE s s0

2+ Rewrite the equation given as v 2 = v0

Take the derivative with respect to s . 2gR 2 dv = 2E ds s

2v

Thus a=v gR 2 dv 2E ds s

Problem 13.65 Suppose that a tunnel could be drilled straight through the earth from the north pole to the south pole and the air evacuated from it. An object dropped from the surface would fall with the acceleration a = gs/RE , where g is the acceleration of gravity at sea level, RE is radius of the earth, and s is the distance of the object from the center of the earth. (The acceleration due to gravitation is equal to zero at the center of the earth and increases linearly with the distance from the center.) What is the magnitude of the velocity of the dropped object when it reaches the center of the earth? Solution: The velocity as a function of position is given by
v dv gs . = ds RE

N Tunnel s RE

Separate variables and integrate: v2 = g RE s 2 + C.

At s = RE , v = 0, from which C = gRE . Combine and reduce: v 2 = gRE 1 s2 2 RE gRE

At the center of the earth s = 0, and the velocity is v =

Problem 13.66 Determine the time in seconds required for the object in Problem 13.65 to fall from the surface of the earth to the center. The earths radius is 6370 km. Solution: From Problem 13.65, the acceleration is
a=v
v 0

g dv = s ds RE
s RE

v du =

g RE

s ds

v2 =

g RE

2 s2) (RE

Recall that v = ds/dt v=


0 RE

ds g = dt RE g RE
tf

2 s2 RE

ds
2 s2 RE

dt
0

g tf = sin1 RE tf =

s RE

0 RE

= sin1 (1)

RE = 1266 s = 21.1 min g 2

Problem 13.67 The coordinates (in meters) of a point moving in the x -y plane are given as functions of time by x = 20t 2 160 and y = t 3 + 40t . Determine the magnitudes of the velocity and acceleration of the point at t = 2 s. Solution:
x = 20t 2 160, vx = 40t, ax = 40 y = t 3 + 40t, vy = 3t 2 + 40, ay = 6t vx (2) = 40(2) = 80 v= vy (2) = 3(2)2 + 40 = 52 ax (2) = 40 ay (2) = 6(2) = 12 a= (40)2 + (12)2 = 41.8 m/s2 (80)2 + (52)2 = 95.4 m/s

Problem 13.68 In terms of a particular reference frame, the position of the center of mass of the F-14 at the time shown (t = 0) is r = 10i + 6j + 22k (m). The velocity from t = 0 to t = 4 s is v = (52 + 6t)i + (12 + t 2 )j (4 + 2t 2 )k (m/s). What is the position of the center of mass of the plane at t = 4 s?

Solution:
r0 = 10i + 6j + 22k m v = (52 + 6t)i + (12 + t 2 )j (4 + 2t 2 )k m/s x4 =
0 4

vx dt = 52t + 3t 2 + x0

x4 = (52)(4) + 3(4)2 + 10 m = 266.0 m y4 =


0 4

vy dt = 12t + t 3 /3 + y0

y4 = 12(4) + (4)3 /3 + 6 m = 75.3 m z4 =


0 4

vz dt = (4t + 2t 3 /3) + z0

z4 = 4(4) 2(4)3 /3 + 22 = 36.7 m r|t =4s = 266i + 75.3j 36.7k (m)

Problem 13.69 The acceleration of an object moving in the x -y plane is a = (4t 2)i + (2t 2 + 4)j (ft/s2 ). At t = 0, its position is r = 3i 2j (ft) and is velocity is v = 6i + 8j (ft/s). What are the position and velocity of the object at t = 3 s? Solution:
ax = 4t 2, vx = 2t 2 2t + 6, sx = 2 3 t t 2 + 6t + 3 3

2 1 ay = 2t 2 + 4, vy = t 3 + 4t + 8, sy = t 4 + 2t 2 + 8t 2 3 6 sx (3) = 2 3 (3) (3)2 + 6(3) + 3 = 30 3

1 sy (3) = (3)4 + 2(3)2 + 8(3) 2 = 26.5 6 r(3) = (30i + 26.5j)ft vx (3) = 2(3)2 2(3) + 6 = 18 2 vy (3) = (3)3 + 4(3) + 8 = 2 3 v(3) = (18i + 2j)ft/s

Problem 13.70 A projectile is launched from ground level with initial velocity v0 = 20 m/s. Determine its range R if (a) 0 = 30 ; (b) 0 = 45 (c) 0 = 60 .

u0 x R

Solution: Set g = 9.81 m/s2 , v0 = 20 m/s


1 ay = g, vy = gt + v0 sin 0 , sy = gt 2 + v0 sin 0 t 2 ax = 0, vx = v0 cos 0 , sx = v0 cos 0 t When it hits the ground, we have 2v0 sin 0 1 0 = gt 2 + v0 sin 0 t t = 2 g R = v0 cos 0 t R = v0 2 sin 20 g

a) b) c)

0 = 30 R = 35.3 m 0 = 45 R = 40.8 m 0 = 60 R = 35.3 m

Problem 13.71 A projectile is launched from ground level with an initial velocity v0 = 20 m/s. What initial angle 0 above the horizontal causes the range R to be a maximum, and what is the maximum range? Solution: From 13.70 we know that
R= v0 2 sin 20 , g

2v0 2 cos 20 dR = = 0 0 = 45 d0 g

R(45 ) =

v0 2 sin(90 ) v0 2 = g g

Putting in the numbers for this problem, 0 = 45 , R = 40.8 m

Problem 13.72 Suppose that you are designing a mortar to launch a rescue line from coast guard vessel to ships in distress. The light line is attached to a weight red by the mortar. Neglect aerodynamic drag and the weight of the line for your preliminary analysis. If you want the line to be able to reach a ship 300 ft away when the mortar is red at 45 above the horizontal, what muzzle velocity is required? Solution: From 13.70 we know that
R= v0 2 sin 20 v0 = g Rg sin 20

y 45
72

v0 =

(300 ft)(32.2 ft/s2 ) = 98.3 ft/s sin(90 )

Problem 13.73 In Problem 13.72, what maximum height above the point where it was red is reached by the weight? Solution: From Problem 13.70 we have
vy = gt + v0 sin 0 , 1 sy = gt 2 + v0 sin 0 t 2

When we reach the maximum height, 0 = gt + v0 sin 0 t = v0 sin 0 g v0 sin 0 g


2

1 1 h = gt 2 + v0 sin 0 t h = g 2 2 h= v0 2 sin2 0 2g

+ v0 sin 0

v0 sin 0 g

Putting in the numbers we have h= (98.3 ft/s)2 sin2 (45 ) 2(32.2 ft/s2 ) = 75 ft

Problem 13.74 When the athlete releases the shot, it is 7 ft above the ground. The horizontal distance the shot travels from the point of release to the point where it hits the ground is 60 ft. What was the initial velocity v0 of the shot?

30

Solution:
ax = 0, vx = v0 cos 30 , sx = v0 cos 30 t ay = 32.2 ft/s2 , vy = (32.2 ft/s2 )t + v0 sin 30 sy = (16.1 ft/s2 )t 2 + v0 sin 30 t + 7 ft When it hits the ground we have 60 ft = v0 cos 30 t 0 = (16.1 ft/s2 )t 2 + v0 sin 30 t + 7 ft Solving simultaneously we nd t = 1.608 s, v0 = 43.08 ft/s

Problem 13.75 A pilot wants to drop survey markers at remote locations in the Australian outback. If he ies at a constant velocity v0 = 40 m/s at altitude h = 30 m and the marker is released with zero velocity relative to the plane, at what horizontal d from the desired impact point should the marker be released?
h

Solution: We want to nd the horizontal distance traveled by the marker before it strikes the ground (y goes to zero for t > 0.)
ax = 0 v x = v x0 x = x0 + vx0 t ay = g vy = vy0 gt y = y0 + vy0 t gt 2 /2

From the problem statement, x0 = 0, vy0 = 0, vx0 = 40 m/s, and y0 = 30 m The equation for y becomes y = 30 (9.81)t 2 /2 Solving with y = 0, we get tf = 2.47 s. Substituting this into the equation for x , we get xf = 40tf = 98.9 m

Problem 13.76 If the pitching wedge the golfer is using gives the ball an initial angle 0 = 50 , what range of velocities v0 will cause the ball to land within 3 ft of the hole? (Assume the hole lies in the plane of the balls trajectory). Strategy: We need to nd the velocities which cause the ball to pass through the points (27,3) ft (3 feet short of the hole) and (33,3) feet (3 feet beyond the hole). Solution: Set the coordinate origin at the point where the golfer strikes the ball. The motion in the horizontal (x ) direction is given by ax = 0, Vx = V0 cos 0 , x = (V0 cos 0 )t . The motion in the vertical (y ) direction is given by
ay = g, Vy = V0 sin 0 gt, y = (V0 sin 0 )t gt 2 . 2

v0 30 ft

3 ft

From the x equation, we can nd the time at which the ball reaches the required value of x (27 or 33 feet). This time is tf = xf /(V0 cos 0 ). We can substitute this information the equation for Y with Yf = 3 ft and solve for V0 . The results are: For hitting (27,3) feet, V0 = 31.2 ft/s. For hitting (33,3) feet, V0 = 34.2 ft/s.

Problem 13.77 A batter strikes a baseball 3 ft above home plate and pops it up. The second baseman catches it 6 ft above second base 3.68 s after it was hit. What was the balls initial velocity, and what was the angle between the balls initial velocity vector and the horizontal? Solution: The equations of motion g = 32.2 ft/s2
ax = 0 vx = v0 cos 0 sx = v0 cos 0 t ay = g vy = gt + v0 sin 0 1 sy = gt 2 + v0 sin 0 t + 3 ft 2

Second base
90 90 ft ft

When the second baseman catches the ball we have 127.3 ft = v0 cos 0 (3.68 s) 1 6 ft = (32.2 ft/s2 )(3.68 s)2 + v0 sin 0 (3.68 s) 2 Solving simultaneously we nd v0 = 70.02 ft/s, 0 = 60.4

Home plate

Problem 13.78 A baseball pitcher releases a fastball with an initial velocity v0 = 90 mi/h. Let be the initial angle of the balls velocity vector above the horizontal. When it is released, the ball is 6 ft above the ground and 58 ft from the batters plate. The batters strike zone extends from 1 ft 10 in. above the ground to 4 ft 6 in. above the ground. Neglecting aerodynamic effects, determine whether the ball will hit the strike zone (a) if = 1 ; (b) if = 2 .

4 ft 6 in 1 ft 10 in 58 ft

Solution: The initial velocity is v0 = 90 mi/h = 132 ft/s. The velocity equations are
(1) dvx = 0, from which vx = v0 cos . dt dvy = g , from which vy = gt + v0 sin . dt dx = v0 cos , from which x(t) = v0 cos t , since the initial dt position is zero. dy = gt + v0 sin , from which dt g y(t) = t 2 + v0 sin t + 6, 2 since the initial position is y(0) = 6 ft. At a distance d = 58 ft, the height is h. The time of passage across the home plate is x(tp ) = d = v0 cos tp , from which tp = d . v0 cos

Substitute: y(tp ) = h = g 2 d v0 cos


2

+ d tan + 6.

For = 1 , h = 3.91 ft , Yes, the pitcher hits the strike zone. For = 2 , h = 4.92 ft No, the pitcher misses the strike zone.

(2)

(3)

(4)

Problem 13.79 In Problem 13.78, assume that pitcher releases the ball at an angle = 1 above horizontal, and determine the range of velocities v0 ft/s) within which he must release the ball to hit strike zone. Solution: From the solution to Problem 13.78,
h= g 2 d v0 cos
2

the the (in the


For h = 4.5, v0 = 146.8 ft/s. For h = 1.833, v0 = 102.2 ft/s. The pitcher will hit the strike zone for velocities of release of 102.2 v0 146.8 ft/s , and a release angle of = 1 . Check: The range of velocities in miles per hour is 69.7 mph v0 100.1 mph, which is within the range of major league pitchers, although the 100 mph upper value is achievable only by a talented few (Nolan Ryan, while with the Houston Astros, would occasionally in a game throw a 105 mph fast ball, as measured by hand held radar from behind the plate).

+ d tan + 6,

where d = 58 ft, and 4.5 h 1.833 ft. Solve for the initial velocity: v0 = gd 2 . (d tan + 6 h)

2 cos2

Problem 13.80 A zoology student is provided with a bow and an arrow tipped with a syringe of sedative and is assigned to measure the temperature of a black rhinoceros (Diceros bicornis ). The range of his bow when it is fully drawn and aimed 45 above the horizontal is 100 m. A truculent rhino charges straight toward him at 30 km/h. If he fully draws his bow and aims 20 above the horizontal, how far away should the rhino be when the student releases the arrow?

20

Solution: The strategy is (a) to determine the range and ight time of the arrow when aimed 20 above the horizontal, (b) to determine the distance traveled by the rhino during this ight time, and then (c) to add this distance to the range of the arrow. Neglect aerodynamic drag on the arrow. The equations for the trajectory are: Denote the constants of integration by Vx , Vy , Cx , Cy , and the velocity of the arrow by VA .
(1) dvx = 0, from which vx = Vx . At t = 0, Vx = VA cos . dt dvy = g , from which vy = gt + Vy . At t = 0, Vy = dt VA sin . dx = vx = VA cos , from which x(t) = VA cos t + Cx . At t = dt 0, x(0) = 0, from which Cx = 0. dy = vy = gt + VA sin , from which dt g y = t 2 + VA sin t + Cy . 2 At t = 0, y = 0, from which Cy = 0. The time of ight is given by g y(tight ) = 0 = tight + VA sin tight , 2 from which tight = 2VA sin . g

(2)

(3)

(4)

The range is given by x(tight ) = R = VA cos tight =


2 cos sin 2VA . g

The maximum range (100 meters) occurs when the arrow is aimed 45 above the horizon. Solve for the arrow velocity: VA = gRmax = 31.3 m/s. The time of ight when the angle is 20 is tight = 2VA sin = 2.18 s, g

and the range is R = VA cos tight = 64.3 m. The speed of the rhino is 30 km/h = 8.33 m/s. The rhino travels a distance d = 8.33(2.18) = 18.2 m. The required range when the arrow is released is d + R = 82.5 m .

Problem 13.81 The crossbar of the goalposts in American football is yc = 10 ft above the ground. To kick a eld goal, the ball must make the ball go between the two uprights supporting the crossbar and be above the crossbar when it does so. Suppose that the kicker attempts a 40-yard eld goal (xc = 120 ft), and kicks the ball with an initial velocity v0 = 70 ft/s and 0 = 40 . By what vertical distance does the ball clear the crossbar? Solution: Set the coordinate origin at the point where the ball is kicked. The x (horizontal) motion of the ball is given by ax = 0, Vx = V0 cos 0 , x = (V0 cos 0 )t . The y motion is given by ay = g , gt 2 Vy = V0 sin 0 gt , y = (V0 sin 0 )t . Set x = xc = 120 ft and 2 nd the time tc at which the ball crossed the plane of the goal posts. Substitute this time into the y equation to nd the y coordinate YB of the ball as it passes over the crossbar. Substituting in the numbers (g = 32.2 ft/s2 ), we get tc = 2.24 s and yB = 20.06 ft. Thus, the ball clears the crossbar by 10.06 feet.

v0
xc

yc

Problem 13.82 An American football quarterback stands at A. At the instant the quarterback throws the football, the receiver is at B running at 20 ft/s toward C , where he catches the ball. The ball is thrown at an angle of 45 above the horizontal, and it is thrown and caught at the same height above the ground. Determine the magnitude of the balls initial velocity and the length of time it is in the air. Solution: Set x as the horizontal motion of the football, y as the vertical motion of the football and z as the horizontal motion of the receiver. Set g = 32.2 ft/s2 , 0 = 45 . We have
az = 0, vz = 20 ft/s, sz = (20 ft/s)t 1 ay = g, vy = gt + v0 sin 0 , sy = gt 2 + v0 sin 0 t 2 ax = 0, vx = v0 cos 0 , sx = v0 cos 0 t When the ball is caught we have sz = (20 ft/s)t 1 0 = gt 2 + v0 sin 0 t 2 sx = v0 cos 0 t sx 2 = sz 2 + (30 ft)2 We can solve these four equations for the four unknowns sx , sz , v0 , t We nd t = 1.67 s, v0 = 38.02 ft/s

Receivers path Path of the ball

90

A B 30 ft

Problem 13.83 The cliff divers of Acapulco, Mexico must time their dives that they enter the water at the crest (high point) of a wave. The crests of the waves are 1 m above the mean water depth h = 4 m. The horizontal velocity of the waves is equal to gh. The divers aiming point is 2 m out from the base of the cliff. Assume that his velocity is horizontal when he begins the dive. (a) (b) What is the magnitude of the drivers velocity when he enters the water? How far from his aiming point must a wave crest be when he dives in order for him to enter the water at the crest?
26 m

1m

Solution:
t = 0, vy = 0, y = 27 m, x0 = 0 ay = g = 9.81 m/s Vy = Vy00 gt y = y0 gt 2/2

h 6.4 m 2m

27 m
y = 1 m at tIMPACT for an ideal dive to hit the crest of the wave t1 = tIMPACT = 2.30 s Vy (t1 ) = 22.59 m/s ax = 0 Vx = Vx0 XI = Vx0 t1 + X0 At impact XI = 8.4 m. For impact to occur as planned, then Vx = 8.4/t1 = 3.65 m/s = constant The velocity at impact is (a) |V | = (Vx )2 + [Vy (t1 )]2 = 22.9 m/s

8.4 m

The wave moves at gh = 6.26 m/s. The wave crest travels 2.30 seconds while the diver is in their s = ght1 = 14.4 m.

Problem 13.84 A projectile is launched at 10 m/s from a sloping surface. The angle = 80 . Determine the range R .

10 m/s 30 a

Solution: Set g = 9.81 m/s2 , v0 = 10 m/s.


The equations of motion are ax = 0, vx = v0 cos(80 30 ), sx = v0 cos 50 t 1 ay = g, vy = gt + v0 sin(80 30 )t, sy = gt 2 + v0 sin 50 t 2 When the projectile hits we have R cos 30 = v0 cos 50 t 1 R sin 30 = gt 2 + v0 sin 50 t 2

t = 2.32 s,

R = 17.21 m

Problem 13.85 A projectile is launched at 100 ft/s at 60 above the horizontal. The surface on which it lands is described by the equation shown. Determine the point of impact. Strategy: Find the equations for the x and y coordinates of the projectile and substitute them into the equation for the surface. Solve for the time of impact then substitute this time back into the equations for the x and y coordinates of the projectile.

100 ft/s

60 x y = 0.001x 2

Solution: The motion in the x direction is ax = 0, vx = V0 cos 0 , x = (V0 cos 0 )t , and the motion in the y direction is given by ay = g , vy = (V0 sin 0 ) gt , y = (V0 sin 0 )t gt 2 /2. We know that V0 = 100 ft/s and 0 = 60 . The equation of the surface upon which the projectile impacts is y = 0.001x 2 . Thus, the time of impact, tI , can be determined by substituting the values of x and y from the motion equations into the equation for the surface. t2 2 . Evaluating Hence, we get (V0 sin 0 )tI g I = 0.001(V0 cos 0 )2 tI 2 with the known values, we get tI = 6.37 s Substituting this value into the motion equations reveals that impact occurs at (x, y) = (318.4, 101.4) ft.

Problem 13.86 At t = 0, a steel ball in a tank of oil is given a horizontal velocity v = 2i (m/s). The components of the balls acceleration in m/s2 are ax = 1.2vx , ay = 8 1.2vy , az = 1.2vz . What is the velocity of the ball at t = 1 s?

Solution: Assume that the effect of gravity is included in the given accelerations. The equations for the path are obtained from:
(1) dvx = ax = 1.2vx . Separate variables and integrate: dt dvx = 1.2 dt, vx from which ln(vx ) = 1.2t + Vx . At t = 0, vx (0) = 2, from which ln vx 2 = 1.2t.

Inverting: vx (t) = 2e1.2t . (2) dvy = ay = 8 1.2vy . Separate variables and integrate: dt dvy = 1.2 dt, 8 + vy 1.2 from which ln 8 + vy 1.2 = 1.2t + Vy .

At t = 0, vy (0) = 0, from ln 1 + 1.2 vy 8 = 1.2t. 8 1.2t 1). (e 1.2

Inverting: vy (t) = (3)

dvz = az = 1.2vz , from which ln(vz ) = 1.2t + Vz . Invert to dt obtain vz (t) = Vz e1.2t . At t = 0, vz (0) = 0, hence Vz = 0 and vz (t) = 0. At t = 1 second, vx (1) = 2e1.2 = 0.6024 m/s , and 8 (1 e1.2 ) = 4.66 m/s , or 12

vy (1) =

v = 0.602i 4.66j (m/s) .

Problem 13.87 In Problem 13.86, what is the position of the ball at t = 1 s relative to its position at t = 0? Solution: Use the solution for the velocity components from Problem 13.86. The equations for the coordinates:
(1) dx = vx = 2e1.2t , from which dt x(t) = 2 e1.2t + Cx . 1.2 (3) At t = 0, y(0) = 0, from which y(t) = 8 1.2 e1.2t 1 +t . 1.2 1.2

Since vz (0) = 0 and z(0) = 0, then z(t) = 0. At t = 1, x(1) = 2 (1 e1.2 ) = 1.165 m . 1.2 e1.2 1 +1 1.2 1.2

At t = 0, x(0) = 0, from which x(t) = dy = dt 2 (1 e1.2t ). 1.2 8 (e1.2t 1), from which 1.2 8 1.2 e1.2t + t + Cy . 1.2

y(1) =

8 1.2

= 2.784 m , or

(2)

r = 1.165i 2.784j (m) .

y(t) =

Problem 13.88 The point P moves along a circular path with radius R . Show that the magnitude of its velocity is |v | = R |d/dt |. Strategy: Use Eqs. 13.23

Solution:
x = R cos y = R sin vx = R sin d dt d dt

vy = R cos |V | =

Vx2 + Vy2 d dt
2 2

|V| =

R 2 sin2

+ R 2 cos2

d dt

|V| =

R2

d dt

(sin2 + cos2 )

|V| = R

d dt

Problem 13.89

If y = 150 mm,

d 2y = 0, what are the magnitudes of the velocity and dt 2 acceleration of point P ?

dy = 300 mm/s, and dt

Solution: The equation for the location of the point P is R 2 =


x2 + y2, from which x = dy dt dy dt
2

(R 2

1 y2) 2

= 0.2598 m, and

y dx = dt x d2x dt 2 = 1 x

= 0.1732 m/s,

y 300 mm

y x2

dx dt

dy dt

y x

d2y dt 2

= 0.4619 m/s2 . The magnitudes are:


2 2

|vP | =

dx dt

dy dt

= 0.3464 m/s

|ap | =

d2x dt 2

d2y dt 2

= 0.4619 m/s2

Problem 13.90 A car travels at a constant speed of 100 km/h on a straight road of increasing grade whose vertical prole can be approximated by the equation shown. When the cars horizontal coordinate is x = 400 m, what is the cars acceleration?

y = 0.0003 x 2

x
dx is positive (car is moving to right in sketch). The acceleration is dt d2x d = dt 2 dt V (2Cx)2 +1 4C 2 V x ((2Cx)2 +
3 1) 2

Solution: Denote C = 0.0003 and V = 100 km/h = 27.78 m/s. The


magnitude of the constant velocity is dy dt
2 2

V =

dx dt

dx dt

The equation for the road is y = Cx 2 from which dy = 2Cx dt dx dt .

= 0.0993 m/s2

d2y = 2C dt 2

dx dt

+ 2Cx

d2x dt 2

= 0.4139 m/s2 , or

Substitute and solve: dx = dt V (2Cx)2 + 1 a = 0.099i + 0.414j(m/s2 ) = 27.01 m/s.

Problem 13.91 Suppose that a projectile has the initial conditions shown in Fig. 13.18. Show that in terms of the x y coordinate system with its origin at the highest point of the trajectory, the equation describing the trajectory is y = g (x )2 . 2 2v0 cos2 0

vx (0) = v0 cos 0 , and vy (0) = v0 sin 0 . The accelerations are ax (t) = 0, ay (t) = g . The path of the projectile in the x , y system is obtained by solving the differential equations subject to the initial conditions: g x(t) = (v0 cos 0 )t, y(t) = t 2 + (v0 sin 0 )t 2 Eliminate t from the equations by substituting t= x v0 cos 0

Solution: The initial conditions are t = 0, x(0) = 0, y(0) = 0,

Substitute xp =

2 cos sin v0 0 0 , g

y =

v 2 sin2 0 v 2 sin2 0 g(x )2 0 x tan 0 + x tan 0 + 0 2 2 2g g 2v0 cos 0


2 sin2 v0 0 . 2g

y =

g (x )2 2 cos2 2v0 0

to obtain y(x) = gx 2 + x tan 0 . 2 cos2 2v0 0

At the peak, dy dx = 0,
peak

from which xp =
2 cos sin v0 0 0 , g 2 sin2 v0 0 . 2g

and

yp =

The primed coordinates: y = y yp reduce: y =

x = x xp . Substitute and

g(x + xp )2 + (x + xp ) tan 0 yp . 2 cos2 2v0 0 g 2 + 2x x ) + (x + x ) tan ((x )2 + xp p p 0 2 cos2 2v0 0


2 sin2 v0 0 . 2g

y =

Problem 13.92 The acceleration components of a point are ax = 4 cos 2t , ay = 4 sin 2t , az = 0. At t = 0, its position and velocity are r = i, v = 2j. Show that (a) the magnitude of the velocity is constant; (b) the velocity and acceleration vectors are perpendicular; (c) the magnitude of the acceleration is constant and points toward the origin; (d) the trajectory of a point is a circle with its center at the origin. Solution: The equations for the path are
(1) dvx = ax = 4 cos(2t), from which vx (t) = 2 sin(2t) + Vx . dt dx At t = 0, vx (0) = 0, from which Vx = 0. = vx = 2 sin(2t), dt from which x(t) = cos(2t) + Cx . At t = 0, x(0) = 1, from which Cx = 0. dvy = ay = 4 sin(2t), from which vy (t) = 2 cos(2t) + Vy . At dt dy t = 0, vy (0) = 2, from which Vy = 0. = vy = 2 cos(2t), dt from which y(t) = sin(2t) + Cy . At t = 0, y(0) = 0, from which Cy = 0. For az = 0 and zero initial conditions, it follows that vz (t) = 0 and z(t) = 0. (a) The magnitude of the velocity is |v| = (2 sin(2t))2 + (2 cos(2t))2 = 2 = const. (b) The velocity is v(t) = i2 sin(2t) + j2 cos(2t). The acceleration is a(t) = i4 cos(2t) j4 sin(2t). If the two are perpendicular, the dot product should vanish: a(t) v(t) = (2 sin(2t))(4 cos(2t)) + (2 cos(2t))(4 sin(2t)) = 0, and it does

(c) The magnitude of the acceleration: |a| = (4 cos(2t))2 + (4 sin(2t))2 = 4 = const .

(2)

The unit vector parallel to the acceleration is e= a = i cos(2t) j sin(2t), |a|

(3)

which always points to the origin. (d) The trajectory path is x(t) = cos(2t) and y(t) = sin(2t). These satisfy the condition for a circle of radius 1: 1 = x2 + y2

Problem 13.93 When an airplane touches down at t = 0, a stationary wheel is subjected to a constant angular acceleration = 110 rad/s2 until t = 1 s. (a) (b) What is the wheels angular velocity at t = 1 s? At t = 0, the angle = 0. Determine in radians and in revolutions at t = 1 s.

Solution:
= 110 rad/s2 = t + 0
2 = (1 2 t ) + 0 t + 0

From the problem statement, 0 = 0 = 0 (a) At t = 1 s, = (110)(1) + 0 = 110 rad/s (b) At t = 1 s, = 110(1)2 /2 = 55 radians (8.75 revolutions)

Problem 13.94 Let L be a line from the center of the earth to a xed point on the equator, and let L0 be a xed reference direction. The gure shows the earth seen from above the North Pole. (a) (b) Is d/dt positive or negative? What is the magnitude of d/dt in rad/s?

L0

Solution:
(a) Positive. The earth rotates such that a point on the equator moves eastward. rad 2 = = 7.27 105 rad/s 24(3600) s

L0

(b)

Actually, earth rotates 2 radians in 23 hours 56 min 4.01 s. Noon to Noon is 24 hrs. (The earth moves about 1 around its orbit each day.)

Problem 13.95 The angular acceleration of the line L relative to the line L0 is given as a function of time by = 2.5 1.2t rad/s2 . At t = 0, = 0 and the angular velocity of L relative to L0 is = 5 rad/s. Determine and at t = 3 s.

u L0

Solution:
= 2.5 1.2t = 2.5t 0.6t 2 + 5 = 1.25t 2 0.2t 3 + 5t (3) = 1.25(3)2 0.2(3)3 + 5(3) = 20.85 rad (3) = 2.5(3) 0.6(3)2 + 5 = 7.1 rad/s

Problem 13.96 The angular acceleration of the line L shown in Problem 13.95 relative to the line L0 is given = 22 rad/s2 where is the angular velocity in rad/s. When = 30 , the angular velocity is 10 rad/s. What is the angular velocity when = 60 ? Strategy: Use the chain rule to write the angular acceleration as d d d d = = . = dt d dt d Solution:
= d = 22 d = 2
10 rad/s

d = = 3

/3 /6

2d

ln

10 rad/s

3 6

= (10 rad/s)e/3 = 3.509 rad/s

Problem 13.97 The stationary astronaut activates hydrogen peroxide jets that give him a constant angular acceleration = 0.4 rad/s2 for 2 s. He then turns off the jets and rotates with constant angular velocity. From the time he activates the jets, how long does it take him to rotate 180 relative to his original position?

Solution: The motion has two parts, the angular acceleration phase and the constant angular velocity phase. Angular Acceleration Phase: 0 = 0, 0 = 0
= d = 0.4 rad/s2 dt
2 0 2 0

2 =

dt = t |2 0 = 0.8 rad/s

2 =

t dt = t 2 /2|2 0 = 0.8 rad

Constant Angular Velocity Phase: = 2 + 2 (t 2) rad We want f = 180 = radians = 0.8 + 0.8(tf 2) Solving tf = + 0.8 s 0.8

tf = 4.93 s

Problem 13.98 The hydroelectric generator is started from rest. Its angular acceleration is given by = 6 0.2 rad/s2 , where is the angular velocity in rad/s. What is the angular velocity of the generator 10 s after it is started?

Solution:
= d = 6 0.2 dt 6 0.2 6
0

d = 6 0.2

10 s

dt
0

5 ln

= 10 = 30(1 e2 )

= 25.9 rad/s

Problem 13.99 The rotor of an electric generator is rotating at 200 rpm when the motor is turned off. Due to frictional effects, the angular deceleration of the rotor after it is turned off is = 0.01 rad/s2 , where is the angular velocity in rad/s. How many revolutions does the rotor turn after the motor is turned off? Solution: The angular velocity at t = 0 is
(0) = 200 rev min 1 min 60 s 2 rad rev = 20.944 rad/s. from which (t) = 2094.4e0.01t + C . Count the angular travel from the time the motor is turned off, from which (0) = 0, and C = 2094.4, and (t) = 2094.4(1 e0.01t ). The rotor comes to rest at a time so great that 0 = lim (20.944e0.01t ) 0.
t

The angular velocity at time t is given by d = = 0.01. dt Separate variables and integrate: d = 0.01 dt, from which ln() = 0.01t + C . Invert to obtain (t) = Ce0.01t . At t = 0, (0) = 20.944 rad/s, from which C = 20.944. The angular travel is d = = 20.944e0.01t , dt

Substitute e0.01t 0 into the angular travel to obtain total = 2094.4 rad revs = 333.33 revs 2 rad

Problem 13.100 The needle of a measuring instrument is connected to a torsional spring that gives it an angular acceleration = 4 rad/s2 , where is the needles angular position in radians relative to a reference direction. The needle is given an angular velocity = 2 rad/s in the position = 0. (a) (b) What is the magnitude of the needles angular velocity when = 30 ? What maximum angle does the needle reach before it rebounds?

Solution:
= d = 4 d
2

d = 4
0

22 2 = 2 2 2 2

= 2 1 2 (a) = 2 1 (/6)2 = 1.704 rad/s Maximum angle means = 0. = 1 rad = 57.3

(b)

Problem 13.101 The angle measures the direction of the unit vector e relative to the x axis. The angular velocity of e is = d/dt = 2 rad/s, constant. Determine the derivative de/dt when = 90 in two ways: (a) (b) Use Eq. (13.33). Express the vector e in terms of its x and y components and take its time derivative of e.

Solution:
(a) de d = n = n dt dt when = 90 , n = i de = 2i rad/s when = 90 dt (b) e = (1) cos i + (1) sin j de = sin dt d dt i + cos d dt j

y n

= d = 2 rad/s dt
x

= 90
x

Evaluating at = 90 de d = i = 2i rad/s dt dt

Problem 13.102 The angle measures the direction of the unit vector e relative to the x axis. The angle is given as a function of time by = 2t 2 rad. What is the vector d e/dt at t = 4 s? Solution: By denition:
de = dt where n = i cos + 2 + j sin + 2 d dt n, The angle is = [mod(2t 2 , 2 )]t =4 = mod(32, 2 ) = 0.5841 rad, where mod(x, y ) (modulus) is a standard function that returns the remainder of division of the rst argument by the second. From which, de dt = 16 i cos 0.5841 + = 8.823i + 13.35j 2 + j sin 0.5841 + 2

t =4

is a unit vector in the direction of positive . The angular rate of change is d dt = [4t ]t =4 = 16 rad/s.

t =4

Problem 13.103 The line OP is of constant length R . The angle = 0 t , where 0 is a constant. (a) (b) dx dy and vy = to deterdt dt mine the velocity of P relative to O . Use Eq. (13.33) to determine the velocity of P relative to O , and conrm that your result agrees with the result of (a). Use the relations vx =

P R

Strategy: In part (b), write the position vector of P relative to O as r = R e where e is a unit vector that points from O toward P . Solution:
(a) The point P is described by P = ix + jy . Take the derivative: dx dP =i dt dt +j dy dt . Note that P = R e, and denition (Eq. (13.33)), de = dt d dt n, de dP =R when R is constant. Use the dt dt

(b)

The coordinates are related to the angle by x = R cos , y = R sin . Take the derivative and note that R is a constant and = 0 t , so that dx d = 0 : = R sin dt dt dy = R cos dt d dt . d dt ,

where n is a unit vector in the direction of positive , (i.e., perpendicular to e). Thus n = i cos + 2 + j sin + . 2

Use the trigonometric sum-of-angles identities to obtain: n = i sin + j cos . Substitute, dP = R0 (i sin(0 t) + j cos(0 t)) dt The results are the same.

Substitute into the derivative of the vector P, dP =R dt d dt (i sin + j cos )

= R0 (i sin(0 t) + j cos(0 t)) which is the velocity of the point P relative to the origin O .

Problem 13.104 The armature of an electric motor rotates with a constant angular velocity of 400 rpm (revolutions per minute).
P

(a) (b)

What is the magnitude of the velocity of point P relative to point O ? What are the normal and tangential components of the acceleration of P relative to O ?

80 mm

Solution:
(a) vP /O = (0.08 m)(400 rpm) 2 rad 1 rev 1 min 60 s = 3.35 m/s

(b)

at = 0, an =

(vP /O )2 = 140.4 m/s2 0.08 m

Problem 13.105 The armature shown in Problem 13.104 starts from rest at t = 0. Its angular acceleration is given as a function of time by = 2t m/s2 . Determine the velocity and acceleration of point P relative to point O in terms of normal and tangential components at t = 10 s. Solution: = 2t, = t 2 , = t 3
vP = (0.08)(10)2 = 8 m/s aP t = (0.08)2(10) = 1.6 m/s2 aP n = (0.08)(10)4 = 800 m/s2 vP = (8et ) m/s2 aP = (1.6et + 800en ) m/s2 1 3

Problem 13.106 Suppose you want to design a medical centrifuge to subject samples to a normal acceleration of 1000 gs. (a) If the distance from the center of the centrifuge to the sample is 300 mm, what speed of rotation in rpm is necessary? (b) If you want the centrifuge to reach its design rpm in 1 min, what constant angular acceleration is necessary?

300 mm

Solution:
(a) The normal acceleration at a constant rotation rate is an = R2 , giving = an = R (1000)9.81 = 180.83 rad/s. 0.3

The speed in rpm is N = rad s 1 rev 2 rad 60 s 1 min = 1730 rpm .

(b)

The angular acceleration is = 180.83 = = 3.01 rad/s2 t 60

Problem 13.107 The medical centrifuge shown in Problem 13.106 starts from rest at t = 0 and is subjected to a constant angular acceleration = 3 rad/s2 . What is the magnitude of the total acceleration to which the samples are subjected at t = 1 s? Solution: = 3, = 3t, = 1.5t 2
at = (0.3 m)(3 rad/s2 ) = 0.9 m/s2 an = (0.3 m)(3 rad/s)2 = 2.7 m/s2 a= (0.9)2 + (2.7)2 m/s2 = 2.85 m/s2

Problem 13.108 The radial distance from the axis of the centrifuge in Fig. 13.38 to the sample container is 8 m. Suppose that you want to subject a sample to a normal acceleration of 100g s. (a) (b) What speed of rotation in rpm is necessary? If you want the centrifuge to reach the rpm determined in part (a) in 2 min, what constant angular acceleration is necessary?

Solution: r = 8 m
(a) aN = 2 r = 981 m/s2 2 = 981/8 rad
2

an 8m

/s2

= 11.07 rad/s = 106 rpm (b) d = , constant dt = 0 + t 0 = 0

11.07 rad/s = (120 s) = 0.0923 rad/s2

Problem 13.109 A powerboat being tested for maneuverability is started from rest at t = 0 and driven in a circular path of 12 m radius. The tangential component of the boats acceleration as a function of time is at = 0.4t m/s2 . (a) (b) What are the boats velocity and acceleration in terms of normal and tangential components at t = 4 s? What distance does the boat move along its circular path from t = 0 to t = 4 s?

Solution:
(a) at = 0.4t m/s2 v = 0.2t 2 m/s At t = 4 s, a = 0.4t et + v 2 /r en a = 1.6et + 0.853en v = 3.2et m/s (b) s = 0.2t 3 /3 s |4s = 4.27 m an = +v 2 /r

Problem 13.110 (a) (b)

The angle = 2t 2 rad.

What are the velocity and acceleration of point P in terms of normal and tangential components at t = 1 s? What distance along the circular path does point P move from t = 0 to t = 1 s?

4m O

Solution:
= 2t 2 d = 4t = dt d2 rad =4 2 = dt 2 s s = r = 4 = 8t 2 vt = 16t m/s v = r = 4(4t) = 16t at = (a) dv = 16 m/s2 dt

(b)

s = R = 8t 2 = 8(1)2 = 8 m

et P eN

O 4m

v = 16(1)et m/s = 16 et (m/s) a = R et + R2 eN a = (4)(4)et + (4)(42 )eN (m/s2 ) a = 16et + 64eN (m/s2 )

Problem 13.111 The angle = 2t 2 rad. What are the velocity and acceleration of point P in terms of normal and tangential components when P has gone one revolution around the circular path starting at t = 0? Solution: From the solution to Problem 13.110,
= 2t 2 rad = 4t rad/s = 4 rad/s2 s = 8t 2 m vt = 16t m/s at = 16 m/s2 We want to know v and a when = 2 . Substituting into the rst eqn, we nd that = 2 when t = t1 = 1.77 seconds. From the solution to Problem 13.110, vt = 16t et and a = R et + R2 eN Substituting in the time t1 , we get vt = 28.4et (m/s) a = 16et + 201.1eN (m/s2 )

Problem 13.112 At the instant shown, the crank AB is rotating with a constant counterclockwise angular velocity of 5000 rpm. Determine the velocity of point B (a) in terms of normal and tangential components; (b) in terms of cartesian components.

Solution:
= (5000 rpm) 2 rad rev min 60 sec = 524 rad/s

(a)

1 ft (524 rad/s)et = (87.3et ) ft/s VB = (2 in.) 12 in. VB = (87.3 ft/s)( cos 45 i sin 45 j) = (61.71i 61.71j) ft/s

45 B A x

(b)

in

Problem 13.113 The crank AB in Problem 13.112 is rotating with a constant counterclockwise angular velocity of 5000 rpm. Determine the acceleration of point B (a) in terms of normal and tangential components; (b) in terms of cartesian components. Solution:
= (5000 rpm) 2 rad rev 1 ft 12 in. min 60 sec = 524 rad/s

at = 0, an = (2 in.)

(524 rad/s)2 = 45,693 ft/s2 45,700

(a)

aP = (45, 700en ) ft/s2 ap = (45,700 ft/s2 )(cos 45 i sin 45 j) = (32, 300i 32, 300j) ft/s2

(b)

Problem 13.114 Suppose that you are standing at point P at 30 north latitude (that is, a point 30 north of the equator). The radius of the earth is RE = 6370 km. What are the magnitudes of your velocity and acceleration relative to a nonrotating reference frame with its origin at the center of the earth?
axis of the earth is r = Re cos 30 . Your velocity is given by |v| = re and your acceleration is
2 r. |a| = e

P RE 30

Equator

Solution: Standing at 30 North latitude, your radius from the spin


N r 30 Re

e =

2 rad = 7.27 105 rad/s. (24 h)(3600 s/h) Hence

e = 7.27 105 rad/s. |v| = 401 m/s |a| = 0.0292 m/s2

Problem 13.115 At the instant shown, the magnitude of the airplanes velocity is 130 m/s, its tangential component of acceleration is at = 4 m/s2 , and the rate of change of its path angle is d/dt = 5 /s. (a) (b) What are the airplanes velocity and acceleration in terms of normal and tangential components? What is the instantaneous radius of curvature of the airplanes path?

Solution:
= (5 /s)

rad 180

rad/s 36

aP t = 4 m/s2 , an = (130 m/s) = 11.34 m/s2

(a)

vp = (130et ) m/s ap = (4et + 11.34en ) m/s2 v2 (130 m/s)2 = = 1490 m an 11.34 m/s2

(b)

Problem 13.116 In the preliminary design of a sunpowered car, a group of engineering students estimates that the cars acceleration will be 0.6 m/s2 . Suppose that the car starts from rest at A and the tangential component of its acceleration is at = 0.6 m/s2 . What are the cars velocity and acceleration in terms of normal and tangential components when it reaches B ? Solution:
at = v dv = 0.6 m/s2 ds
v 0

50 m B A 200 m

vdv =
0

(0.6 m/s2 )ds

v 2 = 2(0.6 m/s2 )s At point B SB = 200 + Thus vB = (18.28et ) m/s aB = (0.6et + 6.68en ) m/s2 50 2 m vB = 18.28 m/s, aBn =
2 vB = 6.68 m/s2 50 m

Problem 13.117 After subjecting the car design described in Problem 13.116 to wind tunnel testing, the students estimate that the tangential component of the cars acceleration will be at = 0.6 0.002v 2 m/s2 , where v is the cars velocity in m/s. If the car starts from rest at A, what are its velocity and acceleration in terms of normal and tangential components when it reaches B ? Solution: At point B SB = 200 +
at = v dv = 0.6 0.002v 2 ds aBn =
vB 0

50 2

m
sB

vdv = 0.6 0.002v 2

ds
0

vB = 14.20 m/s,

2 vB = 4.03 m/s2 50 m

at = 0.6 0.002(14.20 m/s)2 = 0.197 m/s2 Thus vB = (14.20et ) m/s aB = (0.197et + 4.03en ) m/s2

Problem 13.118 Suppose that the tangential component of acceleration of the car described in Problem 13.117 is given in terms of the cars position by at = 0.4 0.001s m/s2 , where s is the distance the car travels along the track from point A. What are the cars velocity and acceleration in terms of normal and tangential components at point B ? Solution:
at = dv dv ds = = 0.4 0.001s m/s2 dt ds dt

v
v 0

dv = 0.4 0.001s ds
SB 0

v dv =

(0.4 0.001s) ds
SB 0

0.001s 2 v2 = 0.4s 2 2

From Fig. P13.116, SB = 200 + 2/4 where = 50 m, so SB = 278.5 m Solving for v , v = 12.05 m/s v = 12.05et (m/s) a = (0.4 0.001sB )et + v 2 / eN (m/s2 ) Solving, a = 0.121et + 2.905eN (m/s2 )

Problem 13.119 A car increases its speed at a constant rate from 40 mi/h at A to 60 mi/h at B . What is the magnitude of its acceleration 2 s after the car passes point A?

120 ft 30 B 30 80 ft 100 ft 80 ft x

Solution: Use the chain rule to obtain


dv = a, v ds where a is constant. Separate variables and integrate: v 2 = 2as + C . At s = 0, v(0) = 40 5280 3600 = 58.67 ft/s,

The velocity is as a function of time is v(t) = v(0) + at = 58.67 + 7.817t ft/s. The distance from A is s(t) = 58.67t + 7.817 2 t . 2

from which C = 3441.78. The acceleration is a= v2 C 2s

The distance traveled from A to B is s = 2(80) + (30) and the speed in (120 + 100) = 275 ft, 180

At a point 2 seconds past A, the distance is s(2) = 132.97 ft, and the velocity is v(2) = 74.3 ft/s. The rst part of the hill ends at 142.83, so that at this point the car is still in the rst part of the hill. The tangential acceleration is at = 7.817 ft/s2 . The normal acceleration is an = v2 (74.3)2 = = 46.0 ft/s2 . R 120

The magnitude of the acceleration is [v(s)]s =275 = 60 5280 3600 = 88 ft/s, |a| = 7.8172 + 46.02 = 46.66 ft/s2

from the constant acceleration is a= (88)2 3441.78 = 7.817 ft/s2 . 2(275)

Note: This is a large accelerationthe driver (and passengers) would no doubt be uncomfortable.

Problem 13.120 The car increases its speed at a constant rate from 40 mi/h at A to 60 mi/h at B . Determine the magnitude of its acceleration when it has traveled along the road a distance (a) 120 ft from A and (b) 160 ft from A. Solution: Use the solution in Problem 13.119.
(a) The velocity at a distance 120 ft from A is v(120) = 2as + C = (2)(7.817)(120) + 3441.78 (b) The velocity at distance 160 ft from A is v(160) = 2(7.817)(160) + C = 77.1 ft/s.

= 72.92 ft/s. At 120 ft the car is in the rst part of the hill. The tangential acceleration is at = 7.817 ft/s2 from Problem 13.119. The normal acceleration is (72.92)2 (v(120))2 = = 44.3 ft/s2 . an = R 120 The magnitude of the acceleration is

At 160 ft the car is on the second part of the hill. The tangential acceleration is unchanged: at = 7.817 ft/s2 . The normal acceleration is an = (v(160))2 5943.1 = = 59.43 ft/s2 . R 100

The magnitude of the acceleration is |a| = 7.8172 + 59.432 = 59.94 ft/s2

[Note: The car will lift off from the road.] |a| = 7.8172 + 44.32 = 45 ft/s
2

Problem 13.121 Astronaut candidates are to be tested in a centrifuge with 10-m radius that rotates in the horizontal plane. Test engineers want to subject the candidates to an acceleration of 5 g s, or ve times the acceleration due to gravity. Earths gravity effectively exerts an acceleration of 1 g in the vertical direction. Determine the angular velocity of the centrifuge in revolutions per second so that the magnitude of the total acceleration is 5 g s. Solution:
an 2 + g 2 = (5g)2 an = an = r2 = an /r 24g

10 m

24(9.81 m/s2 ) = 2.19 rad/s 10m

Problem 13.122 After rst-stage separation and before the second-stage engines have red, a rocket is moving at v = 3000 m/s and the angle between its velocity vector and the vertical is 60 . Because aerodynamic forces are negligible, the rockets acceleration is that due to gravity, which is 9.50 m/s2 at the rockets altitude. Determine: (a) the normal and tangential components of the rockets acceleration; and (b) the instantaneous radius of curvature of the rockets path. Solution: The components of the acceleration are aT = g cos(60 )
toward the rear of the rocket, and aN = g sin(60 ) normal to the axis of the rocket directed 30 degrees away from straight down.

60

The normal acceleration is also given by aN = v 2 /r , where r is the radius of curvature of the path. Substituting g = 9.50 m/s2 and v = 3000 m/s into these relations, we get aT = 4.75 m/s2 , aN = 8.23 m/s2 , and r = 1094 km.

Problem 13.123 The athlete releases the shot with velocity v = 16 m/s at 20 above the horizontal. (a) (b) What are the velocity and acceleration of the shot in terms of normal and tangential components when it is at the highest point of its trajectory? What is the instantaneous radius of curvature of the shots path when it is at the highest point of its trajectory?

20

Solution:
ax = 0 vx = vx0 = 16 cos 20 , vy0 = 16 sin 20 (c) ay = 9.81 m/s
2

|aN | = v 2 /,

= v 2 /|an |

= (15)2 /9.81 = 23.0 m

vy = vy0 9.81t = 5.47 9.81t At highest point, vy = 0 (a) (b) v = 16 cos 20 et = 15.0et (m/s) a = 9.81en (m/s2 )

y et eN x

Problem 13.124 At t = 0, the athlete releases the shot with velocity v = 16 m/s. (a) (b) What are the velocity and acceleration of the shot in terms of normal and tangential components at t = 0.3 s? Use the relation an = v 2 / to determine the instantaneous radius of curvature of the shots path at t = 0.3 s.
|an | = v 2 / = v 2 /|an | = (15.2)2 /9.67 = 24.0 m
y V et en Vx an r Vy x 9.81 m s2

Solution: From the solution to Problem 13.123,


vx = 15.0 m/s vy = 5.47 9.81t m/s v=
2 + v2 vx y

At t = 0.3 s,

v = 15.2 m/s

v = 15.2et (m/s) We have the following geometry From the diagram tan r = vy /vx r = 9.55

|an | = 9.81 cos r = 9.67 m/s2 |at | = 9.81 sin r = 1.63 m/s2 a = 1,63et + 9.67en (m/s2 )
r at

Problem 13.125 At t = 0, the athlete releases the shot with velocity v = 16 m/s. Use Eq. (13.42) to determine the instantaneous radius of curvature of the shots path at t = 0.3 s. Solution:
1+ = dy dx d2y dx 2
2 3/2

We now have y(x) vy dy = 0 9.81 dx v x0 d2y 2 = 9.81/vx 0 dx 2 We also know v0 = 16 m/s and vy0 = v0 sin 20 = 5.47 m/s y0 0 vx0 = v0 cos 20 = 15.04 m/s At t = 0.3 s, x = 4.5 m, tx0 0 dy = 0.168 dx d2y = 0.0434 dx 2 and = 24.0 m x 2 vx 0

From the solution to 13.123, vy = vy0 9.81t, hence y = y0 + vy0 t 9.8/(t 2 /2), Also, vx = vx0 x = x0 + vx0 Hence t = x/vx0 and x 9.81 y = v y0 ( ) v x0 2 x v x0
2

Problem 13.126 The cartesian coordinates of a point moving in the xy -plane are x = 20 + 4t 2 m, y = 10 t 3 m. What is the instantaneous radius of curvature of the path of the point at t = 3 s? Solution: The components of the velocity: v = 8t i (3t 2 )j. At t = 3 seconds, the magnitude of the velocity is |v|t =3 = (8t)2 + (3t 2 )2 = 36.12 m/s. The components of the acceleration are a = 8i (6t)j. The instantaneous path angle is
tan = vy 3t 2 = . vx 8t

At t = 3 seconds, = 0.8442 rad. The unit vector parallel to the path is et = i cos + j sin . The unit vector normal to the path pointing toward the instantaneous radial center is en = i cos 2 + j sin 2 = i sin j cos .

The normal acceleration is the component of acceleration in the direction of en . Thus, an = en a or an = 8 sin + (6t) cos . At t = 3 seconds, an = 5.98 m/s2 . The radius of curvature at t = 3 seconds is = |v|2 = 218 m an

Problem 13.127 The helicopter starts from rest at t = 0. The cartesian components of its acceleration are ax = 0.6t m/s2 and ay = 1.8 0.36t m/s2 . Determine the tangential and normal components of its acceleration at t = 6 s.

Solution: The solution will follow that of Example 13.11, with the time changed to t = 6 s. The helicopter starts from rest (vx , vy ) = (0, 0) at t = 0. Assume that motion starts at the origin (0, 0). The equations for the motion in the x direction are ax = 0.6t m/s2 , vx = 0.3t 2 m/s, x = 0.1t 3 m, and the equations for motion in the y direction are ay = 1.8 0.36t m/s2 , vy = 1.8t 0.18t 2 m/s, and y = 0.9t 2 0.06t 3 m. At t = 6 s, the variables have the values ax = 3.6 m/s2 , ay = 0.36 m/s2 , vx = 10.8 m/s, vy = 4.32 m/s, x = 21.6 m, and y = 19.44 m. The magnitude of the velocity is given by
|v| =
2 + v 2 = 11.63 m/s. vx y

The magnitude of the acceleration is given by |a| =


2 + a 2 = 3.62 m/s2 . ax y

The normal acceleration component is given by aN =


2 = 1.67 m/s2 |a|2 aT

The unit vector in the tangential direction is given by eT = vx i + vy j v = = 0.928i + 0.371j. |v| |v|

The tangential acceleration component is given by aT = a eT = 0.928ax + 0.371ay = 3.21 m/s2 .

Problem 13.128 Use Eq. (13.42) to determine the instantaneous radius of curvature of the path of the helicopter in Problem 13.127 at t = 6 s.
1+ dy dx d2y dx 2
2 3/2

Solution: Equation (13.42) is =

We need y(x)for the path of the helicopter of Example 13.11. We have x = 0.1t 3 and y = 0.9t 2 0.06t 3 . Solving the x relation for t and substituting this into the y relation, we get y = (0.9)(10x)2/3 0.06(10x). The derivatives required to evaluate the radius of curvature are dy = dx 2 (0.9)(10)2/3 x 1/3 0.6, 3 2 (0.9) (10)2/3 x 4/3 . 3 dy = dx

and

d2y 1 = dx 2 3

Substituting these into Equation (13.42), we get x = 21.6 m, 0.4,

d2y = 0.0154, and = 80.96 m. This agrees with the value dx 2 |v|2 = 80.96 m using the values in Problem 13.128 found from = aN

Problem 13.129* For astronaut training, the airplane shown is to achieve weightlessness for a short period of time by ying along a path such that its acceleration is ax = 0 and ay = g . If the velocity of the plane at O at time t = 0 is v = v0 i, show that the autopilot must y the airplane so that its tangential component of the acceleration as a function of time is gt v0 1+ gt v0
2

x O

at = g

Solution: The velocity of the path is v(t) = v0 i gt j. The path


angle is : tan = sin = vy gt = , vx v0 gt
2 v0

+ (gt)2

The unit vector parallel to the velocity vector is e = i cos + j sin . The acceleration vector is a = jg . The component of the acceleration tangent to the ight path is at = g sin ., from which at = g gt
2 v0

+ (gt)2

Divide by v0 ,
1 2 2

at = g 1 +

gt v0

gt v0

Problem 13.130* In Problem 13.129, what is the airplanes normal component of acceleration as a function of time?

Solution: From Problem 13.129, the velocity is v(t) = v0 i gt j.


The ight path angle is , from which cos = v0
2 + (gt)2 v0

The unit vector parallel to the ight path is e = i cos + j sin . The unit vector normal to e is en = i cos 2 + j sin 2

= i sin j cos , pointing toward the instantaneous radial center of the path. The acceleration is a = jg . The component parallel to the normal component is an = g cos , from which
1 2 2

an = g

v0
2 + (gt)2 v0

=g 1+

gt v0

Problem 13.131 and

If y = 100 mm,

d 2y = 0, what are the velocity and acceleration of dt 2 P in terms of normal and tangential components?

dy = 200 mm/s, dt

y 300 mm

Solution: The equation for the circular guide is R 2 = x 2 + y 2 , from which x = R 2 y 2 = 0.283 m, and
dx y = dt x dy = vx = 0.0707 m/s. dt

The velocity of point P is vp = ivx + jvy , from which the velocity is |v| = =
2 + v 2 = 0.212 m/s. The angular velocity vx y

|v| = 0.7071 rad/s. R

The angle is = tan1 ax = y x = 19.5


2

d dvx = dt dt 1 x dy dt

y dy x dt y x2 dx dt dy dt y x d2y dt 2

= 0.1591 m/s2 The unit vector tangent to the path (normal to the radius vector for a circle) is ep = i sin + j cos , from which at = ax sin = 53.0 mm/s2 since ay = 0 an = R2 = 0.150 m/s2 . Check: an = ax cos = 0.15 m/s2 check.

Problem 13.132* Suppose that the point P in Problem 13.131 moves upward in the slot with velocity dy v = 300et (mm/s). When y = 150 mm, what are and dt 2 d y ? dt 2 Solution: The position in the guide slot is y = R sin , from which
= sin1 y R = sin1 (0.5) = 30 . and vx = 150 mm/s (Since the point is moving upward in the slot, vy is positive.). The velocity along the path in the guide slot is assumed constant, hence at = 0. The normal acceleration is an = |v|2 = 300 mm/s2 R

x = R cos = 259.8 mm. From the solution to Problem 13.131, y vx = x dy y = vy . dt x


2 + v2 = v vx y y

directed toward the radius center, from which d2y = an sin = 150 mm/s2 dt 2 y x
2

The velocity is |v| = 300 =


1 2

+ 1, from which

vy = 300

y x

+1

= 259.8 mm/s

Problem 13.133* A car travels at 100 km/h on a straight road of increasing grade whose vertical prole can be approximated by the equation shown. When x = 400 m, what are the tangential and normal components of the cars acceleration? Solution: The strategy is to use the acceleration in cartesian coordinates found in the solution to Problem 13.90, nd the angle with respect to the x -axis,
= tan1 dy dx ,

y y = 0.0003 x 2

and use this angle to transform the accelerations to tangential and normal components. From the solution to Problem 13.90 the accelerations are a = 0.0993i + 0.4139j (m/s2 ). The angle at = tan1 d Cx 2 dx = tan1 (6x 104 )x =400 = 13.5 .

x =400

From trigonometry (see gure) the transformation is at = ax cos + ay sin , an = ax sin + ay cos , from which at = 0.000035 . . . = 0 , an = 0.4256 m/s2 Check: The velocity is constant along the path, so the tangential comdv ponent of the acceleration is zero, at = = 0, check. dt

Problem 13.134 A boy rides a skateboard on the concrete surface of an empty drainage canal described by the equation shown. He starts at y = 20 ft, and the magnitude of his velocity is approximated by v = 2(32.2)(20 y) ft/s. (a) (b) Use Equation 13.42 to determine the instantaneous radius of curvature of the boys path when he reaches the bottom. What is the normal component of his acceleration when he reaches the bottom?

y y = 0.03 x 2

Solution:
(a) y = 0.03x 2 , so dy d2y = 0.06. = 0.06x and dx dx 2 From Eq (13.42), [1 + (0.06x)2 ]3/2 = ft. 0.06 At x = 0, (b) = 16.7 ft.

At the bottom of the canal the values are dx dt dy dt d2y dt 2 = K 20 = 35.89 ft/s. d2x dt 2 dx dt

x =0

x =0

= 0,

x =0 2

= 0,

x =0

= 2C

= 77.28 ft/s2 .
x =0

The angle with respect to the x axis at the bottom of the canal is = tan1
1 1

The magnitude of the velocity is dy dt


2

dy dx

x =0

= 0.

dx dt

= v = K(20 y) 2 = K(20 Cx 2 ) 2 ,

From the solution to Problem 2.133, the tangential and normal accelerations are at = ax cos + ay sin , an = ax sin + ay cos , from which at = 0 , and an = 77.28 ft/s2 .

where K = 8.025, C = 0.03. From y = Cx 2 , dy = 2Cx dt d2y = 2C dt 2 Substitute: K(20 Cx 2 ) 2 dx = . 1 dt (4C 2 x 2 + 1) 2 Since the boy is moving the right, dx > 0, dt and dx dx . = dt dt
1

dx dt dx dt
2

, d2x dt 2 Check: The velocity is constant along the path, so the tangential dv component of the acceleration is zero, at = = 0. check. By dt inspection, the normal acceleration at the bottom of the canal is identical to the y component of the acceleration. check.

+ 2Cx

The acceleration is d2x = dt 2 KCx


1 (20 Cx 2 ) 2 (4C 2 x 2 1 + 1) 2 1

dx dt

K(4C 2 x)(20 Cx 2 ) 2
3 (4C 2 x 2 + 1) 2

dx dt

Problem 13.135 In Problem 13.134, what is the normal component of the boys acceleration when he has passed the bottom and reached y = 10 ft? Solution: Use the results of the solutions to Problems 13.133 and
13.134. From the solution to Problem 13.134, at y = 10 ft, at = ax cos + ay sin , an = ax sin + ay cos , x= dx dt d2x dt 2 y C = 18.257 ft,
1

From the solution to Problem 13.133,

from which
1 y =10

from which = 17.11 ft/s. at = 23.78 ft/s2 , an = 11.84 ft/s2

y =10

= K(20 Cx 2 ) 2 (4C 2 x 2 + 1) 2

y =10

= K

dx dt

y =10

Cx
1 (20 Cx 2 ) 2 (4C 2 x 2

+ 1) 2

1 (4C 2 x)(20 Cx 2 ) 2 + 3 (4C 2 x 2 + 1) 2 y =10 = 24.78 ft/s. d2y dt 2 = 2C dx dt


2

y =10

+ 2Cx

d2x dt 2

= 9.58 ft/s2 .
y =10

The angle is = tan1

dy dx

y =10

= 47.61 .

Problem 13.136* Using Eqs (13.41): (a) Show that the relations between the cartesian unit vectors and the unit vectors et and en are i = cos et sin en and j = sin et + cos en (b) Show that d et /dt = d/dt en and d en /dt = d/dt et .

Solution: Equations (13.41) are et = cos i + sin j and en = sin i + cos j.


(a) Multiplying the equation for et by cos and the equation for en by ( sin ) and adding the two equations, we get i = cos et sin en . Similarly, by multiplying the equation for et by sin and the equation for en by cos and adding, we get j = sin et + cos en . d et = Taking the derivative of et = cos i + sin j, we get dt d d ( sin i + cos j) = en . dt dt Similarly, taking the derivative of en = sin i + cos j, we get d en /dt = (d/dt)et

(b)

Problem 13.137 The polar coordinates of the collar A as functions of time are r = 1 + 0.2t 2 m and = 2t rad. Determine the velocity of the collar in terms of polar coordinates at t = 1 s.

A r

Solution:
r = 1 + 0.2t 2 m = 2t rad r = 0.4t m/s = 2 rad/s e v=r er + r At t = 1 s, r = 1.2 m, and v = 0.4er + 2.4e (m/s) r = 0.4 m/s,

Problem 13.138 In Problem 13.137, what is the acceleration of the collar in terms of polar coordinates at t = 1 s? Solution:
r = 1 + 0.2t 2 m = 2t rad = 2 rad/s r = 0.4t m/s r = 0.4 m/s2 At t = 1 s, =0 r = 1.2 m, r = 0.4 m/s,

= 2 rad/s r = 0.4 m/s2 , and + r ) e 2 )er + (2r a = (r r a = 4.4er + 1.6e (m/s2 )

Problem 13.139 The polar coordinates of point A of the crane are given as functions of time by r = 12 + 0.4t 2 ft and = 0.02t 3 rad. Determine the velocity of A in terms of polar coordinates at t = 2 s.
r

Solution:
r = 12 + 0.4t 2 , r = 0.8t, r = 0.8, = 0.02t 3 = 0.06t 2 = 0.12t

At t = 2 s we have r = 13.6, r = 1.6, r = 0.8, = 0.16 = 0.24 = 0.24

e = (1.6er + 3.26e ) ft/s2 vA = r er + r

Problem 13.140 In Problem 13.139, determine the acceleration of A of the crane in terms of polar coordinates at t = 2 s. Solution: Use the data from problem 13.139
2 )er + (r + 2r )e = (0.0166er + 4.03e ) ft/s2 aA = (r r

Problem 13.141 The radial line rotates with a constant angular velocity of 2 rad/s. Point P moves along the line at a constant speed of 4 m/s. Determine the magnitude of the velocity and acceleration of P when r = 2 m. Solution: The angular velocity of the line is
d = = 2 rad/s, dt from which d2 = 0. dt 2

y 2 rad/s P

4 m/s

O
The magnitude is |v| = 42 + 42 = 5.66 m/s .

The radial velocity of the point is dr = 4 m/s, dt from which d2r = 0. dt 2

The acceleration is a = [2(4)]er + [2(4)(2)]e = 8er + 16e (m/s2 ). The magnitude is e = 4er + 4e (m/s). |a| = 82 + 162 = 17.89 m/s2

The vector velocity is v= dr dt er + r d dt

Problem 13.142 The collar A slides on the vertical bar. At the instant shown, its coordinates are x = 0.9 m, y = 0.6 m, and its velocity and acceleration are v = 4j (m/s) and a = 9.81j (m/s2 ). Determine the velocity of A in terms of polar coordinates.

Solution: Given
x = 0.9, x = 0, x =0 y = 0.6, y = 4, y = 9.81 We have = tan1 Then er = cos i + sin j e = sin i + cos j i = cos er sin e j = sin er + cos e y x = tan1 0.6 0.9 = 33.7

vA = (4j) m/s = 4 m/s(sin er + cos e ) vA = (2.22er + 3.33e ) m/s

Problem 13.143 In Problem 13.142, determine the acceleration of A in terms of polar coordinates. Solution: See Problem 13.143
aA = (9.81 m/s2 )j = (9.81 m/s2 )(sin er + cos e ) aA = (5.44er + 8.16e ) m/s2

Problem 13.144 A boat searching for underwater archaeological sites in the Aegean Sea moves at 4 knots and follows the path r = 10 m, where is in radians. (A knot is one nautical mile, or 1852 meters, per hour.) When = 2 rad, determine the boats velocity (a) in terms of polar coordinates and (b) in terms of cartesian coordinates. Solution: The velocity along the path is
v=4 (a) 1852 m 1 knot 1h 3600 s = 2.06 m/s.

The path is r = 10 . The velocity vr = dr d d = (10) = 10 m/s. dt dt dt

The velocity along the path is related to the components by


2 + v2 = v 2 = vr

dr dt

+ r2

d dt

= 2.062 .

At = 2 , r = 10(2 ) = 62.8 m. Substitute: 2.062 = 10 d dt


2

+ r2

d dt

= (100 + 62.82 )

d dt

from which

d = 0.0323 rad/s, dt v = r d = 2.032 m/s dt

vr = 10 (b)

d = 0.323 m/s , dt

From geometry, the cartesian components are vx = vr cos + v sin , and vy = vr sin + v cos . At = 2 , vx = vr , and vy = v

Problem 13.145 The collar A slides on the circular bar. The radial position of A (in meters) is given as a function of by r = 2 cos . At the instant shown, = 25 and d/dt = 4 rad/s. Determine the velocity of A in terms of polar coordinates.

r u x

Solution:
, r 2 cos 2 r = 2 cos , r = 2 sin = 2 sin Using the given data we have
=0 = 4, = 25 ,

r = 1.813, r = 3.381, r = 29.00 e = (3.381er + 7.25e ) m/s v=r er + r

Problem 13.146 In Problem 13.145, d 2 /dt 2 = 0 at the instant shown. Determine the acceleration of A in terms of polar coordinates. Solution: See problem 13.145
2 )er + (r + 2r )e = (58.0er 27.0e ) m/s2 a = (r r

Problem 13.147 The radial coordinate of the earth satellite is related to its angular position by r= 1.91 107 m. 1 + 0.5 cos

Satellite

The product of the radial position and the transverse component of the velocity is rv = 8.72 1010 m2 /s. What is the satellites velocity in terms of polar coordinates when = 90 ? Solution:
At = 90 , r = 1.91 107 m = p r= p , 1 + 0.5 cos (p)(0.5)( sin ) 2 (1 + 0.5 cos )

r = We also know that

rv = 8.72 1010 m2 /s , hence However v = r = 8.72 1010 m2 /s r 2 , we get Solving for = 0.000239 rad/s and r = 2283 m/s and from above v = 4565 m/s v = 2283er + 4565e (m/s)

Problem 13.148* In Problem 13.147, what is the satellites acceleration in terms of polar coordinates when = 90 ? Solution: Set A = 1.91 107 m, B = 8.72 1010 m2 /s
r= A =B , rv = r(r ) 1 + 0.5 cos B A2 (1 + 0.5 cos )2 B2 A4

= B = r2 = B A2

= (1 + 0.5 cos ) sin

(1 + 0.5 cos )3 sin

r =

0.5A sin = 0.5B sin (1 + 0.5 cos )2 A B2 0.5B cos = 0.5 A A3 cos (1 + 0.5 cos )2

r =

When = 90 we have r = A, r = Thus 2 )er + (r + 2r )e = (1.091er ) m/s2 a = (r r


2 B = B , = B , r = 0, 2A A2 A4

Problem 13.149 A bead slides along a wire that rotates in the xy -plane with constant angular velocity 0 . The radial component of the beads acceleration is zero. The radial component of its velocity is v0 when r = r0 . Determine the polar components of the beads velocity as a function of r . Strategy: The radial component of the beads velocity dr is vr = , and the radial component of the acceleration dt is ar = d 2r r dt 2 d dt
2

r x

dvr dt

2 . r0

By using the chain rule, viz., dvr dvr dr dvr = = vr . dt dr dt dr you can express the radial component of the acceleration dvr 2 in the form ar = vr r0 . dr Solution: From the strategy:
ar = 0 = vr dvr 2 r. 0 dr The transverse component is v = r d dt = r0 , from which

2 rdr , from which Separate variables and integrate: vr dvr = 0 2 vr

v=

2 + 2 (r 2 r 2 )e + r e v0 0 0 0 r

2 = 0

r2 2

+ C.

At r = r0 , vr = v0 , from which C= vr =
2 2 r 2 v0 0 0 , 2

and

2 + 2 (r 2 r 2 ). v0 0 0

Problem 13.150 If the motion of a point in the xy plane is such that its transverse component of acceleration a is zero, show that the product of its radial position and its transverse velocity is constant: rv = constant. Solution: We are given that a = r + 2vr = 0. Multiply the entire relationship by r . We get
0 = (r 2 + 2rvr ) = r 2 d dt + 2r dr dt =
r

d 2 (r ). dt

d 2 (r ) = 0, then r 2 = constant. Now note that v = dt r. We have r 2 = r(r) = rv = constant. This was what we needed to prove. Note that if

Problem 13.151* From astronomical data, Kepler deduced that the line from the sun to a planet traces out equal areas in equal times (Fig. a). Show that this result follows from the fact that the transverse component a of the planets acceleration is zero. [When r changes by an amount dr and changes by an amount d (Fig. b), the resulting differential element of area is given by dA = 1 r(rd )]. 2 Solution: From the solution to Problem 13.150, a = 0 implies
that r 2 = r 2 d = constant. dt

t2 t2 + t A t1 + t

A t1

(a) y

The element of area is dA = 1 r(rd), 2 1 = r 2 = constant. 2

r + dr d dA r

or

1 d dA = r r dt 2 dt

(b)

Thus, if

dA = constant, then equal areas are swept out in equal times. dt

Problem 13.152 The bar rotates in the xy -plane with constant angular velocity 0 . The radial component of acceleration of the collar C is ar = Kr , where K is a constant. When r = r0 , the radial component of velocity of C is v0 . Determine the polar components of the velocity of C as function of r .

r x

Solution: Use the same strategy used in Problem 13.131. The radial acceleration is given by
dvr 2, r0 ar = vr dr where the chain rule has been used to obtain dvr dvr = vr , dt dr from which ar = Kr = vr dvr 2 r. 0 dr

At r = r0 , vr = v0 , from which C= vr =
2 ( 2 K)r 2 v0 0 0 , 2 2 + ( 2 K)(r 2 r 2 ). v0 0 0

and

The transverse velocity is v = 0 r , from which v=


2 + ( 2 K)(r 2 r 2 )e + r e v0 0 0 0 r

Separate variables and integrate:


2 K)rdr, vr dvr = (0 2 2 vr 2 K) r + C. = (0 2 2

from which

Problem 13.153 The hydraulic actuator moves the pin P upward with constant velocity v = 2j (m/s). Determine the velocity of the pin in terms of polar coordinates and the angular velocity of the slotted bar when = 35 . Solution:
vP = 2j (m/s) r = r er e v=r er + r Also, r = 2i + y j (m) v=y j (m/s) = 2j (m/s) r =y sin =y r cos r= x2 + y2 tan = y x Hence

x 2m

e V=r er + r V = 1.15er + 1.64e (m/s)

. y

= 35 , Solving, we get y = 1.40 m, r = 1.15 m/s, r = 2.44 m, = 0.671 rad s

er

Problem 13.154 The hydraulic actuator moves the pin P upward with constant velocity v = 2j (m/s). Determine the acceleration of the pin in terms of polar coordinates and the angular acceleration of the slotted bar when = 35 . Solution: From Problem 13.153
V = 2j m/s, constant a= dv 0 dt = = 0.671 rad/s = 35 2 + 2 sec2 y = 2(2 sec )(sec tan ) 2 = [2 sec tan ]( ) sec = 0.631 rad/s2

= = tan =

y = 2 m/s y =0

y y = x 2

y = 2 tan y = 2 sec2

x = 2m

Problem 13.155 In Example 13.15, determine the velocity of the cam follower when = 135 (a) in terms of polar coordinates and (b) in terms of cartesian coordinates.

Solution:
(a) = 135 , = d/dt = 4rad/s , and = 0. r = 0.15(1 + 0.5 cos )1

r u

Follower x Cam

= 0.232 m. dr d = 0.075 sin (1 + 0.5 cos )2 dt dt = 0.508 m/s. v= dr d er + r e dt dt

= 0.508er + 0.928e (m/s). (b) vx = vr cos v sin = 1.015 m/s. vy = vr sin + v cos = 0.297 m/s.

Problem 13.156* In Example 13.15, determine the acceleration of the cam follower when = 135 (a) in terms of polar coordinates and (b) in terms of cartesian coordinates. Solution: See the solution of Problem 13.155.
(a) d d2r = 0.075 dt 2 dt + 0.075 d dt
2

cos (1 + 0.5 cos )2


2

sin2 (1 + 0.5 cos )3

= 0.1905 m/s2 . a= d2r r dt 2 d dt


2

er + r

d2 dr d e +2 dt 2 dt dt

= 3.52er + 4.06e (m/s2 ). (b) ax = ar cos a sin = 0.381 m/s2 ay = ar sin + a cos = 5.362 m/s2 .

Problem 13.157 In the cam-follower mechanism, the slotted bar rotates with constant angular velocity = 10 rad/s and the radial position of the follower A is determined by the prole of the stationary cam. The path of the follower is described by the polar equation r = 1 + 0.5 cos(2 ) ft. Determine the velocity of the cam follower when = 30 (a) in terms of polar coordinates and (b) in terms of cartesian coordinates.

Solution:
(a) = 30 , = d/dt = 10 rad/s, and = 0. r = 1 + 0.5 cos 2 = 1.25 ft. dr d = sin 2 dt dt = 8.66 ft/s. v= d dr er + r e dt dt

= 8.66er + 12.5e (ft/s). (b) vx = vr cos v sin = 13.75 ft/s, vy = vr sin + v cos = 6.50 ft/s.

Problem 13.158* In Problem 13.157, determine the acceleration of the cam follower when = 30 (a) in terms of polar coordinates and (b) in terms of cartesian coordinates.

Solution: See the solution of Problem 13.157.


(a) d2r = 2 2 cos 2 dt 2 = 100 ft/s2 . a= d2r r dt 2 d dt
2

er + r

d2 dr d e . +2 dt 2 dt dt

= 225er 173e (ft/s2 ). (b) ax = ar cos a sin = 108 ft/s2 , ay = ar sin + a cos = 263 ft/s2 .

Problem 13.159 The cartesian coordinates of a point P in the x y plane are related to its polar coordinates of the point by the equations x = r cos and y = r sin . (a) (b) Show that the unit vectors i, j are related to the unit vectors er , e by i = er cos e sin and j = er sin + e cos . Beginning with the expression for the position vector of P in terms of cartesian coordinates, r = x i + y j, derive Eq. (13.52) for the position vector in terms of polar coordinates. By taking the time derivative of the position vector of point P expressed in terms of cartesian coordinates, derive Eq. (13.47) for the velocity in terms of polar coordinates.

e er r P

(c)

Solution:
(a) From geometry (see Figure), the radial unit vector is er = i cos + j sin , and since the transverse unit vector is at right angles: e = i cos + 2 + j sin + 2 = i sin + j cos .

Solve for i by multiplying er by cos , e by sin , and subtracting the resulting equations: i = er cos e sin . Solve for j by multiplying er by sin , and e by cos , and the results: j = er sin + e cos (b) The position vector is r = x i + y j = (r cos )i + (r sin )j = r(i cos + j sin ). Use the results of Part (a) expressing i, j in terms of er , e : r = r(er cos2 e cos sin + er sin2 + e sin cos ) = r er (c) The time derivatives are: dr dr d =v=i cos r sin dt dt dt +j from which v= d dr (i cos + j sin ) + r (i sin + j cos ). dt dt dr d sin + r cos dt dt ,

Substitute the results of Part (a) v= dr d dr er + r e = er + re dt dt dt

Problem 13.160 The airplane ies in a straight line at 400 mi/h. The radius of its propellor is 5 ft, and the propeller turns at 2000 rpm in the counterclockwise direction when seen from the front of the airplane. Determine the velocity and acceleration of a point on the tip of the propeller in terms of cylindrical coordinates. (Let the z-axis be oriented as shown in the gure.) Solution: The speed is
v = 400 mile h 1h 3600 s 5280 ft 1 mile = 586.7 ft/s The radial acceleration is ar = r2 = 5(209.4)2 = 219324.5 ft/s2 . The transverse acceleration is The angular velocity is = 2000 2 rad 1 rev 1 min 60 s a = r = 209.4 rad/s. dr d2 +2 dt 2 dt d dt = 0,

5 ft z

The radial velocity at the propeller tip is zero. The transverse velocity is v = r = 1047.2 ft/s. The velocity vector in cylindrical coordinates is v = 1047.2e + 586.7ez (ft/s) .

since the propeller rotates at a constant angular velocity. The acceleration az = 0, since the airplane travels at constant speed. Thus a = 219324.5er (ft/s2 )

Problem 13.161 A charged particle P in a magnetic eld moves along the spiral path described by r = 1 m, = 2z rad, where z is in meters. The particle moves along the path in the direction shown with constant speed |v| = 1 km/s. What is the velocity of the particle in terms of cylindrical coordinates?

P z 1 km/s

Solution: The radial velocity is zero, since the path has a constant radius. The magnitude of the velocity is
v= r2 d dt
2

The velocity along the cylindrical axis is dz 1 = dt 2 d dt = 447.2 m/s. v = 894.4e + 447.2ez

dz dt

= 1000 m/s. d dz =2 . dt dt

The velocity vector: The angular velocity is

y
2

Substitute:

v=

r2

d dt

1 4

d dt

d dt

r2 +

1 = 1.25, 4

from which

d 1000 = 894.4 rad/s, = dt 1.25

from which the transverse velocity is v = r d dt = 894.4 m/s.

Problem 13.162 At t = 0, two projectiles A and B are simultaneously launched from O with the initial velocities and elevation angles shown. Determine the velocity of projectile A relative to projectile B (a) at t = 0.5 s and (b) at t = 1 s.

10 m/s A B 60 O 30 x 10 m/s

Solution:
vA = (9.81 m/s2 j)t + (10 m/s)(cos 60 i + sin 60 j ) vB = (9.81 m/s2 j)t + (10 m/s)(cos 30 i + sin 30 j ) vA/B = vA vB = (10 m/s)(0.366i + 0.366j) vA/B = (3.66i + 3.66j) m/s Since vA/B doesnt depend on time, the answer is the same for both times vA/B = (3.66i + 3.66j) m/s

Problem 13.163 Relative to the earth-xed coordinate system, the disk rotates about the xed point O at 10 rad/s. What is the velocity of point A relative to point B at the instant shown?

y A 10 rad/s

B 2 ft O

Solution:
vA = (10 rad/s)(2 ft)i = (20 ft/s)i vB = (10 rad/s)(2 ft)j = (20 ft/s)j vA/B = vA vB = (20i 20j) ft/s

Problem 13.164 Relative to the earth-xed coordinate system, the disk rotates about the xed point O with a constant angular velocity of 10 rad/s. What is the acceleration of point A relative to point B at the instant shown? Solution:
aA = (10 rad/s)2 (2 ft)j = (200 ft/s2 )j aB = (10 rad/s)2 (2 ft)i = (200 ft/s2 )i aA/B = aA aB = (200i 200j) ft/s2

Problem 13.165 The train on the circular track is traveling at 50 ft/s. The train on the straight track is traveling at 20 ft/s. In terms of the earth-xed coordinate system shown, what is the velocity of passenger A relative to passenger B ?

50 0f t

50 ft/s

20 ft/s

Solution:
vA = (20j) ft/s, vB = (50j) ft/s vA/B = vA vB = (70j) ft/s

Problem 13.166 The train on the circular track is traveling at a constant speed of 50 ft/s. The train on the straight track is traveling at 20 ft/s and is increasing its speed at 2 ft/s2 . In terms of the earth-xed coordinate system shown, what is the acceleration of passenger A relative to passenger B ? Solution:
aA = (2j) ft/s2 , aB = (50 ft/s)2 i = (5i) ft/s2 500 ft

aA/B = aA aB = (5i 2j) ft/s2

Problem 13.167 Each bar is 0.4-m long and rotates in the x -y plane. They are connected by a pin at A. Relative to the reference frame shown, bar OA has a counterclockwise angular velocity of 5 rad/s and bar AB has a counterclockwise angular velocity of 10 rad/s. What is the velocity of point B relative to the reference frame at the instant shown? Strategy: Point A moves in a circular path about O . Relative to point A, point B moves in a circular path. (Imagine yourself sitting on point A with point B rotating around you.) Determine the x and y components of the velocity of A relative to O and the velocity of B relative to A. Then you can determine the velocity of B relative to O . Solution:
vB/O = vB/A + vA/O vA/O = (5 rad/s)(0.4 m)( sin 25 i + cos 25 j) = (0.845i + 1.813j) m/s vB/A = (10 rad/s)(0.4 m)( sin 60 i + cos 60 j) = (3.46i + 2.00j) m/s vB/O = (4.31i + 3.81j) m/s

60

25 O x

Problem 13.168 A private pilot wishes to y from a city P to a city Q that is 200 km directly north of city P . The airplane will y with an airspeed of 290 km/h. At the altitude at which the airplane will be ying, there is an east wind (that is, the winds direction is west) with a speed of 50 km/h. What direction should the pilot point the airplane to y directly from city P to city Q? How long will the trip take? Solution: Assume an angle , measured ccw from the east.
VPlane /Ground = VPlane /Air + VAir /Ground VPlane /Air = (290 km/h)(cos i + sin j) VAir /Ground = (50 km/h)i VPlane /Ground = [(290 cos 50)i + (290 sin )j] km/h We want the airplane to travel due north therefore 290 cos 50 = 0 = cos1 Thus the heading is 90 80.07 = 9.93 east of north The ground speed is now v = (290 km/h) sin(80.1 ) = 285.6 km/h The time is t= d 200 km = = 0.700 h = 42.0 min v 285.6 km/h

N W S E 200 km 50 km/h

50 290

= 80.07

Problem 13.169 The river ows north at 3 m/s. (Assume that the current is uniform.) If you want to travel in a straight line from point C to point D in a boat that moves at a constant speed of 10 m/s relative to the water, in what direction should you point the boat? How long does it take to make the crossing?
3 m/s D

Solution: Assume an angle , measured ccw from the east.


VBoat /Ground = VBoat /Water + VWater /Ground VBoat /Water = (10 m/s)(cos i + sin j) VWater /Ground = (3 m/s)j VBoat /Ground = [(10 cos )i + (3 + 10 sin )j] m/s We want the boat to travel at an angle tan = Therefore 400 500

3 + 10 sin 400 = = 25.11 10 cos 500 25.11 north of east

400 m W

N E S

Thus the heading is

The ground speed is now v= (10 cos )2 + (3 + 10 sin )2 = 11.60 m/s

C 500 m

The time is t= d = v 5002 + 4002 m = 55.2 s 11.60 m/s

Problem 13.170 The river ows north at 3 m/s. (Assume that the current is uniform.) What minimum speed must a boat have relative to the water in order to travel in a straight line from point C to point D ? How long does it take to make the crossing?

Solution: Assume an angle , measured ccw from the east.


VBoat /Ground = VBoat /Water + VWater /Ground VBoat /Water = vB /W (cos i + sin j) VWater /Ground = (3 m/s)j VBoat /Ground = [(vB /W cos )i + (3 + vB/W sin )j] m/s We want the boat to travel at an angle tan = Therefore 3 + vB/W sin 500 12 = vB/W = vB/W cos 400 5 cos 4 sin To be a minimum we must have dvB/W 12(4 cos + 5 sin ) 4 = = 0 tan = = 38.7 d (5 cos 4 sin )2 5 Notice, that this heading puts the boats relative velocity perpendicular to the line from C to D . The relative speed is now vB/W = The time is t= d = v 5002 + 4002 m = 342 s 1.874 m/s 12 = 1.874 m/s 5 cos 4 sin 400 500

Problem 13.171 Relative to the earth, the sailboat sails north with speed v0 = 6 knots (nautical miles per hour) and then sails east at the same speed. The tell-tale indicates the direction of the wind relative to the boat. Determine the direction and magnitude of the winds velocity (in knots) relative to the earth.

0 0

60 Tell-tale

N W S E

Solution:
vwind /ground = vwind /boat + vboat /ground In position one we have vwind /ground = vwind /boat1 i + (6 knots)j In position two we have vwind /ground = vwind /boat2 ( cos 60 i + sin 60 j) + (6 knots)i Since the wind has not changed these two expressions must be the same. Therefore vwind /boat 1 = vwind /boat 2 cos 60 + 6 knots 6 knots = vwind /boat 2 sin 60 vwind /boat 1 = 2.536 knots vwind /boat2 = 6.928 knots

Using either position one or position two we have vwind /ground = (2.536i + 6j) knots vwind /ground = (2.536)2 + (6)2 knots = 6.51 knots 2.536 6 = 22.91 east of north

direction = tan1

Problem 13.172 An engineer analyzing a large-scale machining process determines that a tool that moves in a straight line starts from rest at time t = 0 and position s = 0 and moves with acceleration a = 2 + t 1/2 t 3/2 m/s2 from t = 0 to t = 4 s. (a) (b) Draw a graph of the tools position from t = 0 to t = 4 s. What is the maximum velocity of the tool during this time interval, and at what time does it occur?
(a)

Solution:
s0 = v0 = 0

10 8 s, m 6 4

a = 2 + t 1/2 t 3/2 m/s2 v=


0 t

a dt 2 3/2 2 5/2 m/s t t 3 5

2 0 (a) 0 1 2 t, s 3 4

v = 2t + s=
t 0

v dt 2 3 2 5/2 t 5 2 5 2 7/2 t m 7

s = t2 +

Plotting, we get (b) Maximum velocity can be found two way Plotting the velocity or nding where acceleration is zero, calculating the velocity, and checking to see if it is a max or min. The acceleration is a = 2 + t 1/2 t 3/2 m/s Acceleration is zero at t1 = 2.31 s da 3 1 1 1 = t 2 t 2 dt 2 2 Evaluating this at t1 , we get da = 1.95 dt Hence, velocity is a maximum. Evaluating velocity at t = t1 , we get v(t1 ) = 3.72 m/s at t1 = 2.31 s

Problem 13.173 A projectile is launched at 10 m/s from a sloping surface. (a) (b) Determine the value of the angle for which the range R = 15 m. Determine the angle for which the range R is a maximum. What is the maximum range?
30 a

10 m/s

Solution: Set g = 9.81 m/s2 , v0 = 10 m/s


Using horizontal x and vertical y we have ax = 0, vx = v0 cos( 30 ), ay = g vy = gt + v0 sin( 30 )

1 sx = v0 cos( 30 )t, sy = gt 2 + v0 sin( 30 )t 2 When the projectile hits R cos 30 = v0 cos( 30 )t 1 R sin 30 = gt 2 + v0 sin( 30 )t 2 Solving we nd R= 8v0 2 cos( 30 ) sin 3g

(a) Using a root solver we nd 15 = (b) 8v0 2 cos( 30 ) sin = 33.6 or = 86.4 3g

To maximize dR 8v0 2 = cos(30 2) = 0 d 3g The maximum range is therefore R= 8v0 2 2v0 2 cos(30 ) sin(60 ) = = 20.4 m 3g g = 60

Problem 13.174 A catapult designed to throw a line to ships in distress throws a projectile with initial velocity v0 (10.4 sin 0 ), where 0 is the angle above the horizontal. Determine the value of 0 for which the distance the projectile is thrown is a maximum, and show that the maximum distance is
2 0.559v0 . g

0 (1

0.4 sin 0 )

Solution: The path is obtained by integrating the equations of


.6
motion: dvy dvx = g, and = 0, from which dt dt vy (t) = gt + V0 sin 0 , and vx (t) = V0 cos 0 . g y(t) = t 2 + (V0 sin 0 )t , and x(t) = (V0 cos 0 )t. 2 At the end of ight, y(tight ) = 0, from which tight = 2V0 sin 0 . g

Range factor

.5 F .4 a c .3 t o .2 r .1 0 0 10 20 30 40 50 60 70 80 90 Launch angle, deg

The range is R = x(tight ) = 2V02 sin 0 cos 0 . g

Noting V0 = v0 (10.4 sin 0 ), the maximum range occurs when dR d =0= d0 d0 2V02 sin 0 cos 0 g = 0,

from which: after some algebraic reduction, dR = d0 0.8 cos2 sin (1 0.4 sin 0 ) + cos(2) = 0,

from which 0 = 33.41 . This is conrmed as a maximum by graphing g 2 v0

as shown. The maximum range is


2 (1 0.4 sin )2 sin cos v0 0 0 0 g 2 v0 g

Rmax =

= 0.5589
0 =33.41

Problem 13.175 At t = 0, a projectile is located at the origin and has a velocity of 20 m/s at 40 above the horizontal. The prole of the surface the projectile strikes can be approximated by the equation y = 0.4x 0.006x 2 , where x and y are in meters. Determine the approximate coordinates of the point where the projectile hits the ground. Solution: The path of the projectile is obtained by integrating the equations of motion:
d2y = g dt 2 and d2x =0 dt 2 using the initial conditions. The result: g y(t) = t 2 + (V0 sin )t m, 2 and x(t) = (V0 cos )t m. At impact, y(tight ) = yimpact
2 = 0.4ximpact 0.006ximpact .

y 20 m/s y = 0.4x 0.006x 2

40 x Impact point

6 4 2 f(x) 0 2 4

x =29.48

6 25 26 27 28 29 30 31 32 33 34 35 x, meters

Solve for tight : tight = V0 sin g 1 1 2gyimpact V02 sin2

Substitute: ximpact = V02 cos sin g 1 1 2gyimpact V02 sin2

The two functions V02 cos sin 1 f (ximpact ) = g


2 2g(0.4ximpact 0.006ximpact )

ximpact

V02 sin2

were graphed against values of ximpact to determine the zero crossings. Only one crossing is of interest (the other was ximpact = 0), and this value was rened by iteration (using TK Solver Plus) to yield ximpact = 29.477 m from which yimpact = 6.577 m

Problem 13.176 A carpenter working on a house asks his apprentice to throw him an apple. The apple is thrown at 32 ft/s. What two values of 0 will cause the apple to land in the carpenters hand, 12 ft horizontal and 12 ft vertically from the point where it is thrown?

Solution: The path obtained from the equations of motion is


given by g y(t) = t 2 + (V0 sin 0 )t, 2 and x(t) = (V0 cos 0 )t . When the apple reaches the hand g 2 y(tight ) = tight + (V0 sin 0 )tight = 12 ft. 2 Solve for the time of ight: tight = V0 sin 0 g 1 1+ 24g (V0 sin 0 )2

3 2 1 f , 0 h 1 2

Launch angles

3 60 62 64 66 68 70 72 74 76 78 80 angle, deg
,

from which x(tight ) = 12 = V02 sin 0 cos 0 g 1 1 24g (V0 sin 0 )2

The two functions f (0 ) = V02 sin 0 cos 0 g 1 1 24g (V0 sin 0 )2 12

were graphed to determine the zero crossings and the results were rened by iteration (using TK Solver Plus) to obtain 0 = 60.9 and 0 = 74.08

Problem 13.177 A motorcycle starts from rest at t = 0 and moves along a circular track with a 400-m radius. The tangential component of its acceleration is at = 2 + 0.2t m/s2 . When the magnitude of its total acceleration reaches 6 m/s2 , friction can no longer keep the motorcycle on the circular track, and it spins out. How long after it starts does it spin out, and how fast is it going?

en

400 m

et

Solution: The tangential component of the acceleration can be integrated independently of the radial acceleration to determine the angular velocity:
a = r dr d2 +2 dt 2 dt d dt =r d2 = 2 + 0.2t m/s2 , dt 2

since the radius is a constant. From which r = 2t + 0.1t 2 m/s. The radial acceleration is ar = 1 d2r r2 = (2t + 0.1t 2 )2 m/s2 . dt 2 r

10 8 6 4 2 f(t) 0 2 4 6 8 10 10

Zero Crossing

11

12 13 time, s

14

The magnitude of the acceleration is (2t + 0.1t2 )2 r


2

|a| =

(2 + 0.2t)2 +

from which, at time of spin out: 36 = (2 + 0.2t)2 + The function f (t) = (2 + 0.2t)2 + (2t + 0.1t 2 )2 400
2

(2t + 0.1t 2 )2 r

36

was graphed to determine the zero crossing and the result rened by iteration. The result: t = 12.36 s (A zero crossing also occurs at a negative time; it is ignored.) The velocity at spin out is V = [r]t =12.36 s = [2t + 0.1t 2 ]t =12.36 s = 40 m/s V = 40 m s 1 km 103 m 3600 s 1h = 144 km/h

Problem 13.178 At t = 0, a steel ball in a tank of oil is given a horizontal velocity v = 2i m/s. The components of the balls acceleration are ax = cvx , ay = 0.8g cvy , az = cvz , where c is a constant. When the ball hits the bottom of the tank, its position relative to its position at t = 0 is r = 0.8i j (m). What is the value of c?

Solution: Choose a coordinate system with the origin at O and the x axis parallel to the plane surface. The path is obtained by integrating the equations of motion:
dvx = cvx , dt dvy dvz = 0.8g cvy , and = cvz . dt dt

Separating variables and integrating: vx (t) = Cvx ect , vy (t) = From the initial conditions, Cvx = 2, Cvy = vy (t) = 0.8g , Cvz = 0, from which vx = 2ect , c 0.8g + Cvy ect , and vz (t) = Cvz ect . c

.5 .4 .3 .2 .1 f 0 .1 .2 .3 .4 .5

Zero Crossing

.5

1 c, 1/s

1.5

0.8g ct 1), and vz (t) = 0. (e c

Integrating: 0.8g 2 x(t) = ect + Cx , y(t) = c c From the initial conditions, x(0) = 0, y(0) = 0, from which Cx = Cy = y(t) = 2 , c 1 ct e + t + Cy , z(t) = 0. c

0.8g 2 , from which x(t) = (1 ect ) m, c2 c 0.8g 0.8g t m, and z(t) = 0. (1 ect ) c2 c

When the ball strikes the bottom y(t) = 1, x(t) = 0.8. From the equation for x , at time of impact, ect = 1 0.4c, or ct = ln(1 0.4c). Substitute these into the equation for y to obtain 0 = (0.8g)(0.4c + ln(1 0.4c)) + c2 . A graph of f (c) = 0.8g(0.4c + ln(1 0.4c)) + c2 was used to nd the zero crossing, and the result rened by iteration to obtain c = 1.313 s1

Problem 13.179 The polar coordinates of a point P moving in the xy -plane are r = t 3 4t m, = t 2 t rad. (a) Draw a graph of the magnitude of the velocity of P from t = 0 to t = 2 s. (b) Estimate the minimum magnitude of the velocity and the time at which it occurs. Solution:
(a) The velocity is v= dr er + r dt d dt e = (3t2 4)er + (t3 4t)(2t 1)e .

Magnitude of Velocity 10 9 8 7 6 5 |v | 4 3 2 1 0

The magnitude is |vp | = (3t2 4)2 + (t3 4t)2 (2t 1)2 m/s.

The graph of the magnitude is shown. (b) The minimum magnitude is found from a search of the tabulated values of the magnitude: |vp | = 2.63 m/s at t = 0.79 s

.5

1 time, s

1.5

Problem 13.180 (a) Draw a graph of the magnitude of the acceleration of the point P in Problem 13.179 from t = 0 to t = 2 s. (b) Estimate the minimum magnitude of the acceleration and the time at which it occurs. Solution: The acceleration is
a= d2r dt 2 r d dt
2

er + r

d2 dt 2

+2

dr dt

d dt

e .

From Problem 13.197, r = (t 3 4t), dr d2r = 3t 2 4, 2 = 6t ; dt dt

= t 2 t, Substitute:

d d2 = 2t 1, 2 = 2. dt dt

Acceleration 50 45 40 35 30 |a| 25 Minimum = 2.074 @ 0.31 s 20 15 10 5 0 0 .5 1 time, s 1.5 2

ar = 6t (2t 1)2 (t 3 4t), a = (t 3 4t)(2) + 2(3t 2 4)(2t 1). The magnitude is |a| =
2 + a2 . ar

The graph is shown. The minimum is found from a search of the tabulated values of the magnitude of the acceleration: |a|min = 2.0738 m/s2 at t = 0.310 s Note for users of TK Solver Plus.

Problem 13.181 The robot is programmed so that point P describes the path r = 1 0.5 cos 2 t m, = 0.5 0.2 sin[2(t 0.1)] rad. Determine the values of r and at which the magnitude of the velocity of P attains its maximum value.

Solution: The velocity is


vp = dr er + r dt d dt e dr = sin(2 t) m, dt

d = 0.4 cos[2(t 0.1)] dt The magnitude of the velocity is |vp | = ( sin(2 t))2 + (1 0.5 cos(2 t))2 (0.4 cos[2(t 0.1)])2 .

4 3.5 3 2.5 |v | 2 1.5 1 .5 0

Robot hand Velocity

Maximum = 3.248 at 0.73 s 0 .1 .2 .3 .4 .5 .6 .7 .8 .9 1 time, s

The maximum value was found from a search of the tabulated values: |vp |max = 3.248 m/s. The corresponding values of r and are r = 1.066 m and = 37

Problem 13.182 In Problem 13.181, determine the values of r and at which the magnitude of the acceleration of P attains its maximum value. Solution: The acceleration components are:
ar = d2r r dt 2 d dt
2

Robot arm acceleration 25 22.5 20 17.5 15 |a| 12.5 10 7.5 5 2.5 0

, from which:

ar = 2 2 cos(2 t) (1 0.5 cos(2 t))(0.4 cos(2(t 0.1)))2 a = r d2 dt 2 dr +2 dt d dt , from which:

Max = 22.636 at t = 0.475 s

a = (1 0.5 cos(2 t))(0.8 2 sin(2(t 0.1)) + 2( sin(2 t)) (0.4 cos(2(t 0.1))).
2 + a 2 . Although not required by the probThe magnitude is |ap | = ar lem, a graph of the acceleration as a function of time is shown. The magnitude is found by a search of the tabulated values: |ap |max = 22.64 m/s2 . The values of the radius and angle are r = 1.494 m and = 20.5 .

.2

.4 .6 time, s

.8

Problem 13.183 In the cam-follower mechanism, the slotted bar rotates with a constant angular velocity = 10 rad/s, and the radial position of the follower A is determined by the prole of the stationary cam. The path of the follower is described by the polar equation r = 1 + 0.5 cos 2 . (a) (b) Draw a graph of the magnitude of the followers acceleration as a function of for 0 360 . Use your graph to estimate the maximum magnitude of the followers acceleration and the angle(s) at which it occurs.

Solution: Refer to the solutions of Problems 13.157 and 13.158. In these problems, the acceleration components for the follower were derived as functions of the angle . We proceed as in Example 13.15.
(a) We nd the value for = vr = r dr dr dr d dr by = = = 0.5(2) sin(2) dt dt d dt d
360 340

We can now solve for the accelerations and the magnitude is


2 + a 2 . Using an automatic numerical solver given as |a| = ar to plot the results, we get the graph at the right.

(b)

From the graph, the maximum magnitudes of acceleration occur at = 0 and = 180 , and the value for the maximum is |a |MAX = 350 ft/s2
Follower Accel. Magnitude (ft/s^2) vs Angle (deg)

= sin(2). = constant and = 0. Hence, Note that = v = vr er + re = sin(2)er + re . In order to determine the components of the acceleration in polar coordinates, we need to be able to determine all of the variables 2 and that a = r + 2r . r in the right hand sides of ar = r . Since is constant, We already know everything except r and = = 0. We need only to nd r . To nd r , we start with r = vr . Taking a derivative, we get r = r = d d , or sin(2) (vr ) = dt dt d d d [vr ] = [ sin(2)] = 2 cos(2)2 . d dt d

a 320 m 300 a g 280 260 f 240 t 220 s 200 180 s 160 140 0 50 100 150 200 250 Follower Angle (deg) 300 350 400

Problem 13.184 Suppose you throw a ball straight up at 10 m/s and release it at 2 m above the ground. (a) What maximum height above the ground does the ball reach? (b) How long after release it does the ball hit the ground? (c) What is the magnitude of its velocity just before it hits the ground? Solution: The equations of motion for the ball are
ay = g = 9.81 m/s2 , vy = vy 0 gt = 10 9.81t (m/s), and y = y0 + vy 0 t gt 2 /2 = 2 + 10t 9.81t 2 /2 (m). (a) The maximum height occurs when the velocity is zero. Call this time t = t1 . It can be obtained by setting velocity to zero, i.e., vy = 0 = 10 9.81t1 (m/s). Solving, we get t1 = 1.02 s. Substituting this time into the y equation, we get a maximum height of yMAX = 7.10 m. (b) The ball hits the ground when y = 0 m. To nd out when this occurs, we set y = 0 m into the y equation and solve for the time(s) when this occurs. There will be two times, one positive and one negative. Only the positive time has meaning for us. Let this time be t = t2 . The equation for t2 is y = 0 = 2 + 10t2 9.81t 2 2 /2 (m). Solving, we get t2 = 2.22 s.

(c) The velocity at impact is determined by substituting t2 = 2.22 s into the equation for vy . Doing this, we nd that at impact, vy = 11.8 m/s

Problem 13.185 Suppose that you must determine the duration of the yellow light at a highway intersection. Assume that cars will be approaching the intersection traveling as fast as 65 mi/h, that the drivers reaction times are as long as 0.5 s, and that cars can safely achieve a deceleration of at least 0.4g . (a) (b) How long must the light remain yellow to allow drivers to come to a stop safely before the light turns red? What is the minimum distance cars must be from the intersection when the light turns yellow to come to a stop safely at the intersection?
(b) d2s = 0.4g . From which dt 2 The distance traveled after brake application is traveled from brake application to full stop is given by
2 + V t , from which s(t ) = 353.14 ft. s(t)0 = 0.2gt0 0 0 0

Solution: The speed-time equation from initial speed to stop is


given by integrating the equation

ds = 0.4gt + V0 , and s(t) = 0.2gt 2 + V0 t, dt where V0 is the initial speed and the distance is referenced from the point where the brakes are applied. The initial speed is: V0 (a) 65 mi 1h 5280 ft 1 mi 1h 3600 s = 95.33 ft/s.

The distance traveled during the reaction time is d = V0 (0.5) = 95.33(0.5) = 47.66 ft, from which the total distance is dt = 353.14 + 47.66 = 400.8 ft

The time required to come to a full stop ds(t0 ) V0 95.33 = 0 is t0 = = = 7.40 s. dt 0.4g (0.4)(32.17) The drivers reaction time increases this by 0.5 second, hence the total time to stop after observing the yellow light is T = t0 + 0.5 = 7.90 s

Problem 13.186 The acceleration of a point moving along a straight line is a = 4t + 2 m/s2 . When t = 2 s, its position is s = 36 m, and when t = 4 seconds, its position is s = 90 meters. What is its velocity when t = 4 s? Solution: The position-time equation is given by integrating
ds d2s = 4t + 2, from which = 2t 2 + 2t + V0 , and dt 2 dt s(t) = 2 3 t + t 2 + V0 t + d0 , 3 from which (2) 4V0 + d0 = 94 . 3

Subtract (1) from (2) to obtain V0 = 94 80 6 = 2.33 m/s.

where V0 , d0 are the initial velocity and position. From the problem conditions: s(2) = 2 3 2 + (22 ) + V0 (2) + d0 = 36, 3

The velocity at t = 4 seconds is ds(t) dt = [2t 2 + 2t + V0 ]t =4 = 32 + 8 + 2.33 = 42.33 m/s

from which (1) 2V0 + d0 = 80 .s(4) = 3 2 3 4 + (42 ) + V0 (4) + d0 = 90, 3

t =4

Problem 13.187 A model rocket takes off straight up. Its acceleration during the 2 s its motor burns is 25 m/s2 . Neglect aerodynamic drag, and determine (a) (b) the maximum velocity of the rocket during the ight and the maximum altitude the rocket reaches.

Solution: The strategy is to solve the equations of motion for the two phases of the ight: during burn 0 t 2 s seconds, and after burnout: t > 2 s.
Phase 1: The acceleration is: ds d2s = 25t, and s(t) = 12.5t 2 , = 25, from which dt 2 dt since the initial velocity and position are zero. The velocity at burnout is Vburnout = (25)(2) = 50 m/s. The altitude at burnout is hburnout = (12.5)(4) = 50 m. Phase 2. The acceleration is: ds d2s = g(t 2) + Vburnout (t 2), and = g, from which dt 2 dt s(t) = g(t 2)2 /2 + Vburnout (t 2) + hburnout , (t 2). The velocity during phase 1 is constantly increasing because of the rockets positive acceleration. Maximum occurs at burnout because after burnout, the rocket has negative acceleration and velocity constantly decreases until it reaches zero at maximum altitude. The velocity from maximum altitude to impact must be constantly increasing since the rocket is falling straight down under the action of gravity. Thus the maximum velocity during phase 2 occurs when the rocket impacts the ground. The issue of maximum velocity becomes this: is the velocity at burnout greater or less than the velocity at ground impact? The time of ight is given by 0 = g(tight 2)2 /2 + Vburnout (tight 2) + hburnout , from which, in canonical form: (tight 2)2 + 2b(tight 2) + c = 0, where b = (Vburnout /g) and c = (2hburnout /g). The solution (tight 2) = b b2 c = 11.11, = 0.92 s. Since the negative time is not allowed, the time of ight is tight = 13.11 s. The velocity at impact is Vimpact = g(tight 2) + Vburnout = 59 m/s which is higher in magnitude than the velocity at burnout. The time of maximum altitude is given by ds = 0 = g(tmax alt 2) + Vburnout , dt tmax alt 2 = Vburnout = 5.1 s, g from which from which

tmax alt = 7.1 s. The maximum altitude is g hmax = (tmax alt 2)2 + Vburnout (tmax alt 2) + hburnout = 177.42 m 2

Problem 13.188 In Problem 13.187, if the rockets parachute fails to open, what is the total time of ight from takeoff until the rocket hits the ground? Solution: The solution to Problem 13.187 was (serendipitously) posed in a manner to yield the time of ight as a peripheral answer. The time of ight is given there as tight = 13.11 s

Problem 13.189 The acceleration of a point moving along a straight line is a = cv 3 , where c is a constant. If the velocity of the point is v0 , what distance does the v0 point move before its velocity decreases to ? 2 Solution: The acceleration is
dv = cv 3 . Using the chain rule, dt

dv dv ds dv = =v = cv 3 . Separating variables and integrating: dt ds dt ds dv 1 = cds , from which = cs + C . At s = 0, v = v0 , from v2 v 1 v0 v0 1 which = cs , and v = . Invert: v0 cs = 1. v v0 1 + v0 cs v v0 1 When v = , s= 2 cv0

Problem 13.190 Water leaves the nozzle at 20 above the horizontal and strikes the wall at the point indicated. What was the velocity of the water as it left the nozzle? Strategy: Determine the motion of the water by treating each particle of water as a projectile.
20 20 ft 12 ft

35 ft

Solution: Denote = 20 . The path is obtained by integrating the


equations: dvy dvx = g and = 0, dt dt

Substitute: 0= g 2 35 Vn cos
2

+ 35 tan 8,

from which

dy dx = gt + Vn sin , = Vn cos . dt dt g y = t 2 + (Vn sin )t + y0 . 2 x = (Vn cos )t + x0 . Choose the origin at the nozzle so that y0 = 0, and x0 = 0. When the stream is y(timpact ) = (20 12) = 8 ft, the time is g 0 = (timpact )2 + (Vn sin )timpact 8. 2 At this same time the horizontal distance is x(timpact ) = 35 = (Vn cos )timpact , from which timpact = 35 . Vn cos

from which Vn =

35 cos

g = 68.62 ft/s 2(35 tan 8)

Problem 13.191 In practice, a quarterback throws the football with a velocity v0 at 45 above the horizontal. At the same instant, a receiver standing 20 ft in front of him starts running straight down eld at 10 ft/s and catches the ball. Assume that the ball is thrown and caught at the same height above the ground. What is the velocity v0 ?

45 10 ft/s

20 ft

Solution: Denote = 45 . The path is determined by integrating


the equations;

v0
d2x d2y = g, 2 = 0, dt 2 dt from which

45 10 ft/s

dy dx = gt + v0 sin , = v0 cos . dt dt g y = t 2 + (v0 sin )t, 2 x = (v0 cos )t, where the origin is taken at the point where the ball leaves the quarterbacks hand. When the ball reaches the receivers hands, y = 0, from which tight = 2v0 sin . g

20 ft Zero crossing 100 80 60 40 20 f(v) 0 20 40 60 80 100 35 35.5 36 36.5 37 37.5 38 Velocity, ft /s

At this time the distance down eld is the distance to the receiver: x = 10tight + 20. x = (v0 cos )tight , tight = 20 . (v0 cos 10) But also from which

Substitute: 20 = (v0 cos 10) 2v0 sin , g from which

400 g = 2v0 sin (v0 cos 10)2 . The function f (v0 ) = 2v0 sin (v0 cos 10)2 400 g was graphed to nd the zero crossing, and the result rened by iteration: v0 = 36.48 ft/s . Check : The time of ight is t = 1.27 s and the distance down eld that the quarterback throws the ball is d = 12.7 + 20 = 32.7 ft = 10.6y ds , which seem reasonable for a short, lob pass. check.

Problem 13.192 The constant velocity v = 2 m/s. What are the magnitudes of the velocity and acceleration of point P when x = 0.25 m?

y y = 0.2 sin x P x

1m

Solution: Let x = 2t m/s. Then x = 0.25 m at t = 0.125 s. We know that vx = 2 m/s and ax = 0.
From y = 0.2 sin(2 t), we obtain

dy = 0.4 cos(2 t) and dt d2y = 0.8 2 sin(2 t). dt 2 At t = 0.125 s, y = 0.141 m and dy = vy = 0.889 m/s and dt d2y = ay = 5.58 m/s. dt 2 Therefore |v| = |a| =
2 + v 2 = 2.19 m/s, vx y

2 + a 2 = 5.58 m/s2 . ax y

Problem 13.193 The constant velocity v = 2 m/s. What is the acceleration of point P in terms of normal and tangential components when x = 0.25 m? Solution: See the solution of Problem 13.192. The angle
between the x axis and the path is = arctan vy vx = arctan 0.889 2 = 24.0 . Then

ay at

ax

at = ax cos + ay sin = 0 + (5.58) sin 24.0 = 2.27 m/s2 , aN = ax sin ay cos = 0 (5.58) cos 24.0 = 5.10 m/s2 . The instantaneous radius is =
2 + v2 vx y

aN

aN

(2)2 + (0.889)2 = 0.939 m. 5.10

Problem 13.194 The constant velocity v = 2 m/s. What is the acceleration of point P in terms of polar coordinates when x = 0.25 m? Solution: See the solution of Problem 13.192. The polar angle is
= arctan y x = arctan 0.141 0.25 = 29.5 . Then

ay a ar

ar = ax cos + ay sin = 0 + (5.58) sin 29.5 = 2.75 m/s2 , a = ax sin + ay cos = 0 + (5.58) cos 29.5 = 4.86 m/s .
2

ax

Problem 13.195 A point P moves along the spiral path r = (0.1) ft, where is in radians. The angular position = 2t rad, where t is in seconds, and r = 0 at t = 0. Determine the magnitudes of the velocity and acceleration of P at t = 1 s.

P r

Solution: The path r = 0.2t ft, = 2t rad. The velocity components are vr = dr d = 0.2 ft/s, v = r = (0.2t)2 = 0.4t. dt dt

At t = 1 seconds the magnitude of the velocity is |v| =


2 + v2 = vr

0.22 + 0.42 = 0.447 ft/s

The acceleration components are: ar = d2r r dt 2 d2 dt 2 d dt +2


2

= (0.2t)(22 ) ft/s2 , dr dt d dt

a = r

= 2(0.2)(2) = 0.8 ft/s2 .

The magnitude of the acceleration is |a| =

2 + a 2 = 1.13 ft/s2 ar

Problem 13.196 In the cam-follower mechanism, the slotted bar rotates with a constant angular velocity = 12 rad/s, and the radial position of the follower A is determined by the prole of the stationary cam. The slotted bar is pinned a distance h = 0.2 m to the left of the center of the circular cam. The follower moves in a circular path of 0.42 m radius. Determine the velocity of the follower when = 40 (a) in terms of polar coordinates, and (b) in terms of cartesian coordinates.

Solution:
(a) The rst step is to get an equation for the path of the follower in terms of the angle . This can be most easily done by referring to the diagram at the right. Using the law of cosines, we can write R 2 = h2 + r 2 2hr cos . This can be rewritten as r 2 2hr cos + (h2 R 2 ) = 0. We need to nd the components of . We can differentiate and v = r the velocity. These are vr = r the relation derived from the law of cosines to get r . Carrying sin = 0. out this differentiation, we get 2r r 2hr cos + 2hr Solving for r , we get r = sin hr . (h cos r)

P r

R c

and substituting in the numerical values, Recalling that = i.e., R = 0.42 m, h = 0.2 m, = 12 rad/s, and = 40 , we get r = 0.553 m, vr = 2.13 m/s, and v = 6.64 m/s (b) The transformation to cartesian coordinates can be derived from er = cos i + sin j, and e = sin i + cos j. Substituting these into v = vr er + v e , we get v = (vr cos v sin )i + (vr sin + v cos )j. Substituting in the numbers, v = 5.90i + 3.71j (m/s)

Problem 13.197* In Problem 13.196, determine the acceleration of the follower when = 40 (a) in terms of polar coordinates and (b) in terms of cartesian coordinates. Solution:
(a) Information from the solution to Problem 13.196 will be used in this solution. In order to determine the components of the acceleration in polar coordinates, we need to be able to determine 2 and r all of the variables in the right hand sides of ar = r + 2r . We already know everything except r and that a = r . Since is constant, = = 0. We need only to nd the value for r and the value for r at = 40 . Substituting into the original equation for r , we nd that r = 0.553 m at this position on the cam. To nd r , we start with r = vr . Taking a derivative, sin = 0 from Problem 13.196 we start with r r hr cos + hr (we divided through by 2). Taking a derivative with respect to time, we get r = sin + hr 2 cos + hr sin r 2 + 2hr . (h cos r) Evaluating, we get r = 46.17 m/s2 . Substituting this into the equation for ar and evaluating an , we get ar = 125.81 m/s2 and a = 51.2 m/s2 (b) The transformation of cartesian coordinates can be derived from er = cos i + sin j, and e = sin i + cos j. Substituting these into a = ar er + ae e , we get a = ar er + a e , we get a = (ar cos a sin )i + (ar sin + a cos )j. Substituting in the numbers, we get a = 63.46 i 120.1 j(m/s2 ).

Вам также может понравиться